Что первым делается вычитание или деление: Порядок действий в Математике — РОСТОВСКИЙ ЦЕНТР ПОМОЩИ ДЕТЯМ № 7

Содержание

Порядок действий в Математике — РОСТОВСКИЙ ЦЕНТР ПОМОЩИ ДЕТЯМ № 7

Содержание

Порядок действий в Математике

Основные операции в математике

Основные операции, которые используют в математике — это сложение, вычитание, умножение и деление. Помимо этих операций есть ещё операции отношения, такие как равно (=), больше (>), меньше (<), больше или равно (≥), меньше или равно (≤), не равно (≠).

Операции действия:

  • сложение (+)
  • вычитание (-)
  • умножение (*)
  • деление (:)

Операции отношения:

  • равно (=)
  • больше (>)
  • меньше (<)
  • больше или равно (≥)
  • меньше или равно (≤)
  • не равно (≠)

Сложение — операция, которая позволяет объединить два слагаемых.

  • Запись сложения: 5 + 1 = 6, где 5 и 1 — слагаемые, 6 — сумма.

Вычитание — действие, обратное сложению.

  • Запись вычитания: 10 — 1 = 9, где 10 — уменьшаемое, 1 — вычитаемое, 9 — разность.

Если разность 9, сложить с вычитаемым 1, то получится уменьшаемое 10. Операция сложения 9 + 1 = 10 является контрольной проверкой вычитания 10 — 1 = 9.

Умножение — арифметическое действие в виде краткой записи суммы одинаковых слагаемых.

  • Запись: 3 * 4 = 12, где 3 — множимое, 4 — множитель, 12 — произведение.
  • 3 * 4 = 3 + 3 + 3 + 3

В случае, если множимое и множитель поменять ролями, произведение остается одним и тем же. Например: 5 * 2 = 5 + 5 = 10.

Поэтому и множитель, и множимое называют сомножителями.

Деление — арифметическое действие обратное умножению.

  • Запись: 30 : 6 = 5 или 30/6 = 5, где 30 — делимое, 6 — делитель, 5 — частное.

В этом случае произведение делителя 6 и частного 5, в качестве проверки, дает делимое 30.

Если в результате операции деления, частное является не целым числом, то его можно представить в виде дроби.

Возведение степень — операция умножения числа на самого себя несколько раз. 4 = 81 — возведение числа 3 в четвертую степень дает 81 (проверка извлечения корня).

  • 2√16 = 4 — корень второй степени называется — квадратным.
  • При знаке квадратного корня показатель корня принято опускать: √16 = 4.

    3√8 = 2 — корень третьей степени называется — кубическим.

    Сложение и вычитание, умножение и деление, возведение в степень и извлечение корня попарно представляют обратные друг другу действия. Далее узнаем порядок выполнения арифметических действий.

     

    Порядок вычисления простых выражений

    Есть однозначное правило, которое определяет порядок выполнения действий в выражениях без скобок:

    • действия выполняются по порядку слева направо
    • сначала выполняется умножение и деление, а затем — сложение и вычитание.

    Из этого правила становится яснее, какое действие выполняется первым. Универсального ответа нет, нужно анализировать каждый пример и подбирать ход решения самостоятельно.

    Что первое, умножение или деление? — По порядку слева направо.

    Сначала умножение или сложение? — Умножаем, потом складываем.

    Порядок выполнения действий в математике (слева направо) можно объяснить тем, что в нашей культуре принято вести записи слева направо. А необходимость сначала умножить или разделить объясняется самой сутью этих операций.

    Рассмотрим порядок арифметических действий в примерах.

    Пример 1. Выполнить вычисление: 11- 2 + 5.

    Как решаем:

    В нашем выражении нет скобок, умножение и деление отсутствуют, поэтому выполняем все действия в указанном порядке. Сначала вычтем два из одиннадцати, затем прибавим к остатку пять и в итоге получим четырнадцать.

    Вот запись всего решения: 11- 2 + 5 = 9 + 5 = 14.

    Ответ: 14.

    Пример 2. В каком порядке выполнить вычисления в выражении: 10 : 2 * 7 : 5?

    Как рассуждаем:

    Чтобы не ошибиться, перечитаем правило для выражений без скобок. У нас есть только умножение и деление — значит сохраняем записанный порядок вычислений и считаем последовательно слева направо.

    Сначала выполняем деление десяти на два, результат умножаем на семь и получившееся в число делим на пять.

    Запись всего решения выглядит так: 10 : 2 * 7 : 5 = 5 * 7 : 5 = 35 : 5 = 7.

    Ответ: 7.

    Пока новые знания не стали привычными, чтобы не перепутать последовательность действий при вычислении значения выражения, удобно над знаками арифметический действий расставить цифры, которые соответствуют порядку их выполнения.

    Например, в такой последовательности можно решить пример по действиям:


    Действия первой и второй ступени

    В некоторых учебниках по математике можно встретить разделение арифметических действий на действия первой и второй ступени.

    • Действиями первой ступени называют сложение и вычитание, а умножение и деление — действиями второй ступени.

    С этими терминами правило определения порядка выполнения действий звучит так:

    Если выражение не содержит скобок, то по порядку слева направо сначала выполняются действия второй ступени (умножение и деление), затем — действия первой ступени (сложение и вычитание).


    Порядок вычислений в выражениях со скобками

    Иногда выражения могут содержать скобки, которые подсказывают порядок выполнения математических действий. В этом случае правило звучит так:

    Сначала выполнить действия в скобках, при этом также по порядку слева направо выполняется умножение и деление, затем — сложение и вычитание.

    Выражения в скобках рассматриваются как составные части исходного выражения. В них сохраняется уже известный нам порядок выполнения действий.

    Рассмотрим порядок выполнения действий на примерах со скобками.

    Пример 1. Вычислить: 10 + (8 — 2 * 3) * (12 — 4) : 2.

    Как правильно решить пример:

    Выражение содержит скобки, поэтому сначала выполним действия в выражениях, которые заключены в эти скобки.

    Начнем с первого 8 — 2 * 3. Что сначала, умножение или вычитание? Мы уже знаем правильный ответ: умножение, затем вычитание. Получается так:

    8 — 2 * 3 = 8 — 6 = 2.

    Переходим ко второму выражению в скобках 12 — 4. Здесь только одно действие – вычитание, выполняем: 12 — 4 = 8.

    Подставляем полученные значения в исходное выражение:

    10 + (8 — 2 * 3) * (12 — 4) : 2 = 10 + 2 * 8 : 2.

    Порядок действий: умножение, деление, и только потом — сложение. Получится:

    10 + 2 * 8 : 2 = 10 + 16 : 2 = 10 + 8 = 18.

    На этом все действия выполнены.

    Ответ: 10 + (8 — 2 * 3) * (12 — 4) : 2 = 18.

    Можно встретить выражения, которые содержат скобки в скобках. Для их решения, нужно последовательно применять правило выполнения действий в выражениях со скобками. Удобнее всего начинать выполнение действий с внутренних скобок и продвигаться к внешним. Покажем на примере.

    Пример 2. Выполнить действия в выражении: 9 + (5 + 1 + 4 * (2 + 3)).

    Как решаем:

    Перед нами выражение со скобками. Это значит, что выполнение действий нужно начать с выражения в скобках, то есть, с 5 + 1 + 4 * (2 + 3). Но! Это выражение также содержит скобки, поэтому начнем сначала с действий в них:

    2 + 3 = 5.

    Подставим найденное значение: 5 + 1 + 4 * 5. В этом выражении сначала выполняем умножение, затем — сложение:

    5 + 1 + 4 * 5 = 5 + 1 + 20 = 26.

    Исходное значение, после подстановки примет вид 9 + 24, и остается лишь выполнить сложение: 9 + 26 = 35.

    Ответ: 4 + (3 + 1 + 4 * (2 + 3)) = 35.

     

    Порядок вычисления в выражениях со степенями, корнями, логарифмами и иными функциями

    Если в выражение входят степени, корни, логарифмы, синус, косинус, тангенс и котангенс, а также другие функции — их значения нужно вычислить до выполнения остальных действий. При этом важно учитывать правила из предыдущих пунктов, которые задают очередность действий в математике.

    Другими словами, перечисленные функции по степени важности можно приравнивать к выражению в скобках.

    И, как всегда, рассмотрим, как это работает на примере.

    Пример 1. Вычислить (4 + 1) * 3 + 62 : 3 — 7.

    Как решаем:

    В этом выражении есть степень 62. И нам нужно найти ее значение до выполнения остальных действий. Выполним возведение в степень: 62 = 36.

    Подставляем полученное значение в исходное выражение:

    (4 + 1) * 3 + 36 : 3 — 7.

    Дальше нам уже все знакомо: выполняем действия в скобках, далее по порядку слева направо выполняем сначала умножение, деление, а затем — сложение и вычитание. Ход решения выглядит так:

    (4 + 1) * 3 + 36 : 3 — 7 = 3 * 3 + 36 : 3 — 7 = 9 + 12 — 7 = 14.

    Ответ: (3 + 1) * 2 + 62 : 3 — 7 = 14.

    У нас есть статья «знаки больше, меньше или равно», она может быть полезной для тебя!

    Еще больше практики — в детской школе Skysmart. Ученики занимаются на интерактивной платформе, в комфортном темпе и с поддержкой внимательных учителей.

    Чтобы ребенок занимался математикой в удовольствие и чувствовал себя увереннее в школе, запишите его на бесплатный вводный урок.

    Познакомим с форматом и вдохновим на учебу!

    Что сперва умножение или деление

    Вот вам очень про­стой мате­ма­ти­че­ский при­мер:

    8 / 2(2 + 2)

    Вы уди­ви­тесь, но боль­шин­ство людей не смо­гут пра­виль­но это посчи­тать. Посчи­тай­те сами и потом смот­ри­те пра­виль­ный ответ:

    В интер­не­те мно­го спо­ров про такие при­ме­ры, поэто­му мы реши­ли разо­брать­ся, какие ошиб­ки совер­ша­ют чаще все­го и поче­му мно­гие счи­та­ют непра­виль­но. Для реше­ния нам пона­до­бят­ся три мате­ма­ти­че­ских пра­ви­ла:

    1. То, что в скоб­ках, выпол­ня­ет­ся в первую оче­редь. Если ско­бок несколь­ко, они выпол­ня­ют­ся сле­ва напра­во.
    2. При отсут­ствии ско­бок мате­ма­ти­че­ские дей­ствия выпол­ня­ют­ся сле­ва напра­во, сна­ча­ла умно­же­ние и деле­ние, потом — сло­же­ние и вычи­та­ние.
    3. Меж­ду мно­жи­те­лем и скоб­кой (или дву­мя скоб­ка­ми) может опус­кать­ся знак умно­же­ния.

    Раз­бе­рём подроб­нее, что это зна­чит в нашем слу­чае.

    1. То, что в скоб­ках, выпол­ня­ет­ся в первую оче­редь. То есть в нашем при­ме­ре, вне зави­си­мо­сти от чего угод­но, сна­ча­ла схлоп­нут­ся скоб­ки:

    8 / 2(2 + 2) → 8 / 2(4)

    2. Меж­ду чис­лом и скоб­кой мож­но опу­стить знак умно­же­ния. У нас перед скоб­кой двой­ка, то есть мож­но сде­лать такую заме­ну:

    3. Мате­ма­ти­че­ские дей­ствия при отсут­ствии ско­бок выпол­ня­ют­ся сле­ва напра­во: как при чте­нии, сна­ча­ла умно­же­ние и деле­ние, потом — сло­же­ние и вычи­та­ние. Умно­же­ние и деле­ние име­ют оди­на­ко­вый при­о­ри­тет. Нет тако­го, что сна­ча­ла все­гда дела­ет­ся умно­же­ние, затем деле­ние, или наобо­рот. Со сло­же­ни­ем и вычи­та­ни­ем то же самое.

    Неко­то­рые счи­та­ют, что раз мно­жи­те­ли были напи­са­ны близ­ко друг к дру­гу (когда там сто­я­ли скоб­ки), то оно выпол­ня­ет­ся в первую оче­редь, ссы­ла­ясь при этом на раз­ные мето­ди­че­ские посо­бия. На самом деле это не так, и нет тако­го скры­то­го умно­же­ния, кото­рое име­ет при­о­ри­тет над дру­гим умно­же­ни­ем или деле­ни­ем. Это такое же умно­же­ние, как и осталь­ные, и оно дела­ет­ся в общем поряд­ке — как и при­ня­то во всём мате­ма­ти­че­ском мире.

    Полу­ча­ет­ся, что нам сна­ча­ла надо сло­жить 2 + 2 в скоб­ках, потом 8 раз­де­лить на 2, и полу­чен­ный резуль­тат умно­жить на то, что в скоб­ках:

    8 / 2 × (2 + 2) = 8 / 2 × 4 = 4 × 4 = 16

    Кста­ти, если на айфоне запи­сать это выра­же­ние точ­но так же, как в усло­вии, теле­фон тоже даст пра­виль­ный ответ.

    А инже­нер­ный каль­ку­ля­тор на Windows 10 так запи­сы­вать не уме­ет и про­пус­ка­ет первую двойку-множитель. Попро­буй­те сами &#128578;

    Тут в тред вры­ва­ют­ся мате­ма­ти­ки и с воп­ля­ми «Шустеф!» пояс­ня­ют кри­ком:

    «В АЛГЕБРЕ ТОТ ЖЕ ПОРЯДОК ДЕЙСТВИЙ, ЧТО И В АРИФМЕТИКЕ, но есть исклю­че­ние: в алгеб­ре знак умно­же­ния свя­зы­ва­ет ком­по­нен­ты дей­ствия силь­нее, чем знак деле­ния, поэто­му знак умно­же­ния опус­ка­ет­ся. Напри­мер, a:b·c= a: (b·c)».

    Этот текст из «Мето­ди­ки пре­по­да­ва­ния алгеб­ры», курс лек­ций, Шустеф М. Ф., 1967 год. (стр. 43)

    Раз в спор­ном при­ме­ре знак умно­же­ния опу­щен, то спор­ный при­мер алгеб­ра­и­че­ский, а зна­чит, сна­ча­ла умно­жа­ем 2 на 4, а потом 8 делим на 8!

    А вот как на это отве­ча­ют те, кто дей­стви­тель­но в теме и не ленит­ся пол­но­стью посмот­реть пер­во­ис­точ­ник:

    «Для устра­не­ния недо­ра­зу­ме­ний В. Л. Гон­ча­ров ука­зы­ва­ет, что пред­по­чти­тель­нее поль­зо­вать­ся в каче­стве зна­ка деле­ния чер­той и ста­вить скоб­ки [87]. П. С. Алек­сан­дров и А. Н. Кол­мо­го­ров [59] пред­ло­жи­ли изме­нить поря­док дей­ствий в ариф­ме­ти­ке и решать, напри­мер, так: 80:20×2=80:40=2 вме­сто обыч­но­го: 80:20×2=4×2=8. Одна­ко это пред­ло­же­ние не нашло под­держ­ки».

    Если апел­ли­ро­вать к Фри­де Мак­совне Шустеф, то выхо­дит, что:

    1. В. Л. Гон­ча­ров гово­рит так: «Ребя­та, исполь­зуй­те чер­ту и ставь­те скоб­ки, что­бы ни у кого не было вопро­сов про при­о­ри­тет».
    2. Если у нас всё же бит­ва ариф­ме­ти­ки и алгеб­ры, то, по П. С. Алек­сан­дро­ву и А. Н. Кол­мо­го­ро­ву, при­мер нуж­но решать сле­ва напра­во, как обыч­но. Они, конеч­но, пред­ло­жи­ли решать такое по-другому, но науч­ное сооб­ще­ство их не под­дер­жа­ло.

    Самое инте­рес­ное, что даль­ше в при­ме­рах Фри­да Мак­сов­на поль­зу­ет­ся как раз пра­виль­ным поряд­ком дей­ствий, объ­яс­няя реше­ние. Даже там, где есть умно­же­ние на скоб­ку с опу­щен­ным зна­ком, она выпол­ня­ет дей­ствия сле­ва напра­во.

    На данном уроке подробно рассмотрен порядок выполнения арифметических действий в выражениях без скобок и со скобками. Учащимся предоставляется возможность в ходе выполнения заданий определить, зависит ли значение выражений от порядка выполнения арифметических действий, узнать отличается ли порядок арифметических действий в выражениях без скобок и со скобками, потренироваться в применении изученного правила, найти и исправить ошибки, допущенные при определении порядка действий.

    Наблюдение за изменением значения выражения от порядка выполнения арифметических действий

    В жизни мы постоянно выполняем какие-либо действия: гуляем, учимся, читаем, пишем, считаем, улыбаемся, ссоримся и миримся. Эти действия мы выполняем в разном порядке. Иногда их можно поменять местами, а иногда нет. Например, собираясь утром в школу, можно сначала сделать зарядку, затем заправить постель, а можно наоборот. Но нельзя сначала уйти в школу, а потом надеть одежду.

    А в математике обязательно ли выполнять арифметические действия в определенном порядке?

    Сравним выражения:
    8-3+4 и 8-3+4

    Видим, что оба выражения совершенно одинаковы.

    Выполним действия в одном выражения слева направо, а в другом справа налево. Числами можно проставить порядок выполнения действий (рис. 1).

    Рис. 1. Порядок действий

    В первом выражении мы сначала выполним действие вычитания, а затем к результату прибавим число 4.

    Во втором выражении сначала найдем значение суммы, а потом из 8 вычтем полученный результат 7.

    Видим, что значения выражений получаются разные.

    Сделаем вывод: порядок выполнения арифметических действий менять нельзя.

    Порядок выполнения арифметических действий в выражениях без скобок

    Узнаем правило выполнения арифметических действий в выражениях без скобок.

    Если в выражение без скобок входят только сложение и вычитание или только умножение и деление, то действия выполняют в том порядке, в каком они написаны.

    В этом выражении имеются только действия сложения и вычитания. Эти действия называют действиями первой ступени.

    Выполняем действия слева направо по порядку (рис. 2).

    Рис. 2. Порядок действий

    Рассмотрим второе выражение

    В этом выражении имеются только действия умножения и деления – это действия второй ступени.

    Выполняем действия слева направо по порядку (рис. 3).

    Рис. 3. Порядок действий

    В каком порядке выполняются арифметические действия, если в выражении имеются не только действия сложения и вычитания, но и умножения и деления?

    Если в выражение без скобок входят не только действия сложения и вычитания, но и умножения и деления, или оба этих действия, то сначала выполняют по порядку (слева направо) умножение и деление, а затем сложение и вычитание.

    Рассуждаем так. В этом выражении имеются действия сложения и вычитания, умножения и деления. Действуем по правилу. Сначала выполняем по порядку (слева направо) умножение и деление, а затем сложение и вычитание. Расставим порядок действий.

    Вычислим значение выражения.

    Порядок выполнения арифметических действий в выражениях со скобками

    В каком порядке выполняются арифметические действия, если в выражении имеются скобки?

    Если в выражении имеются скобки, то сначала вычисляют значение выражений в скобках.

    Мы видим, что в этом выражении имеется действие в скобках, значит, это действие выполним первым, затем по порядку умножение и сложение. Расставим порядок действий.

    Вычислим значение выражения.

    Правило выполнения арифметических действий в выражениях без скобок и со скобками

    Как нужно рассуждать, чтобы правильно установить порядок арифметических действий в числовом выражении?

    Прежде чем приступить к вычислениям, надо рассмотреть выражение (выяснить, есть ли в нём скобки, какие действия в нём имеются) и только после этого выполнять действия в следующем порядке:

    1. действия, записанные в скобках;

    2. умножение и деление;

    3. сложение и вычитание.

    Схема поможет запомнить это несложное правило (рис. 4).

    Рис. 4. Порядок действий

    Выполнение тренировочных заданий на изученное правило

    Рассмотрим выражения, установим порядок действий и выполним вычисления.

    Будем действовать по правилу. В выражении 43 – (20 – 7) +15 имеются действия в скобках, а также действия сложения и вычитания. Установим порядок действий. Первым действием выполним действие в скобках, а затем по порядку слева направо вычитание и сложение.

    43 – (20 – 7) +15 =43 – 13 +15 = 30 + 15 = 45

    В выражении 32 + 9 * (19 – 16) имеются действия в скобках, а также действия умножения и сложения. По правилу первым выполним действие в скобках, затем умножение (число 9 умножаем на результат, полученный при вычитании) и сложение.

    32 + 9 * (19 – 16) =32 + 9 * 3 = 32 + 27 = 59

    В выражении 2*9-18:3 отсутствуют скобки, зато имеются действия умножения, деления и вычитания. Действуем по правилу. Сначала выполним слева направо умножение и деление, а затем от результата, полученного при умножении, вычтем результат, полученный при делении. То есть первое действие – умножение, второе – деление, третье – вычитание.

    Узнаем, правильно ли определен порядок действий в следующих выражениях.

    В этом выражении скобки отсутствуют, значит, сначала выполняем слева направо умножение или деление, затем сложение или вычитание. В данном выражении первое действие – деление, второе – умножение. Третье действие должно быть сложение, четвертое – вычитание. Вывод: порядок действий определен верно.

    Найдем значение данного выражения.

    Во втором выражении имеются скобки, значит, сначала выполняем действие в скобках, затем слева направо умножение или деление, сложение или вычитание. Проверяем: первое действие – в скобках, второе – деление, третье – сложение. Вывод: порядок действий определен неверно. Исправим ошибки, найдем значение выражения.

    В этом выражении также имеются скобки, значит, сначала выполняем действие в скобках, затем слева направо умножение или деление, сложение или вычитание. Проверяем: первое действие – в скобках, второе – умножение, третье – вычитание. Вывод: порядок действий определен неверно. Исправим ошибки, найдем значение выражения.

    Расставим порядок действий в выражении, используя изученное правило (рис. 5).

    Рис. 5. Порядок действий

    Мы не видим числовых значений, поэтому не сможем найти значение выражений, однако потренируемся применять изученное правило.

    Действуем по алгоритму.

    В первом выражении имеются скобки, значит, первое действие в скобках. Затем слева направо умножение и деление, потом слева направо вычитание и сложение.

    Во втором выражении также имеются скобки, значит, первое действие выполняем в скобках. После этого слева направо умножение и деление, после этого – вычитание.

    Проверим себя (рис. 6).

    Рис. 6. Порядок действий

    Сегодня на уроке мы познакомились с правилом порядка выполнения действий в выражениях без скобок и со скобками.

    Список литературы

    1. М. И. Моро, М.А. Бантова и др. Математика: Учебник. 3 класс: в 2-х частях, часть 1. – М.: «Просвещение», 2012.
    2. М.И. Моро, М.А. Бантова и др. Математика: Учебник. 3 класс: в 2-х частях, часть 2. – М.: «Просвещение», 2012.
    3. М.И. Моро. Уроки математики: Методические рекомендации для учителя. 3 класс. – М.: Просвещение, 2012.
    4. Нормативно-правовой документ. Контроль и оценка результатов обучения. – М.: «Просвещение», 2011.
    5. «Школа России»: Программы для начальной школы. – М.: «Просвещение», 2011.
    6. С.И. Волкова. Математика: Проверочные работы. 3 класс. – М.: Просвещение, 2012.
    7. В.Н. Рудницкая. Тесты. – М.: «Экзамен», 2012.

    Дополнительные рекомендованные ссылки на ресурсы сети Интернет

    Домашнее задание

    1. Определи порядок действий в данных выражениях. Найди значение выражений.

    2. Определи, в каком выражении такой порядок выполнения действий:

    1. умножение; 2. деление;. 3. сложение; 4. вычитание; 5. сложение. Найди значение данного выражения.

    3. Составь три выражения, в которых такой порядок выполнения действий:

    1. умножение; 2. сложение; 3. вычитание

    1. сложение; 2. вычитание; 3. сложение

    1. умножение; 2. деление; 3. сложение

    Найди значение этих выражений.

    Если вы нашли ошибку или неработающую ссылку, пожалуйста, сообщите нам – сделайте свой вклад в развитие проекта.

    Если несколько действий выполняются одно за другим, то результат, зависит от порядка действий.

    Если производить действия в порядке их записи.

    Если же сначала сложить 2 и 1 и вычесть полученную сумму из 4, то получим 1.

    Чтобы указать, в каком порядке нужно выполнять действия (в тех случаях, когда результат зависит от порядка действий), пользуются скобками. Действия, заключенные в скобки, выполняются раньше других. В нашем случае:

    Чтобы не загромождать чрезмерно записи, условились не писать скобок:

    1. в том случае, когда действия сложения и вычитания, следуя друг за другом, должны выполняться в том порядке, в каком они записаны;
    2. в том случае, когда внутри скобок производятся действия умножения или деления; например, вместо 2 + (4 · 5) = 22 пишут 2 + 4 · 5 = 22.

    При вычислении таких выражений, которые либо совсем не содержат скобок, либо содержат лишь такие скобки, внутри которых больше нет скобок, нужно производить действия в таком порядке:

    1. сначала выполняются действия, заключенные в скобки; при этом умножение и деление делаются в порядке из следования, но раньше, чем сложение и вычитание;
    2. затем выполняются остающиеся действия, причем опять умножение и деление делаются в порядке из следования, но раньше сложения и вычитания.

    Сначала выполняем умножения:
    2 · 5 = 10
    3 · 3 = 9
    затем вычитание:
    10 – 9 = 1

    Сначала выполняем действия в скобках:
    16 – 2 · 7 + 4 = 16 – 14 + 4 = 6
    2 + 5 = 7

    Теперь выполняем остающиеся действия:
    9 + 16 : 4 – 2 · 6 + 6 · 7 =
    = 9 + 4 – 12 + 42 =
    = 43

    Часто для указания порядка действий необходимо заключать в скобки такие выражения, которые сами уже содержат скобки. Тогда, кроме обычных (круглых), применяют скобки иной формы, например квадратные []. Если в скобки нужно заключить выражение, содержащее уже круглые и квадратные скобки, пользуются фигурными скобками <>. Вычисление подобных выражений производится в следующем порядке: сначала производятся вычисления внутри всех круглых скобок в вышеуказанной последовательности. Затем — вычисления внутри всех квадратных скобок по тем же правилам. Далее — вычисления внутри фигурных скобок и т.д.. Наконец, выполняются остающиеся действия.

    Выполняем действия в круглых скобках, имеем:
    8 – 6 = 2
    10 – 2 · 3 = 10 – 6 = 4

    действия в квадратных скобках дают:
    14 – 3 · 2 = 8

    выполняя остающиеся действия скобках находим:
    5 + 2 · 8 + 32 : 4 = 5 + 16 + 8 = 29

    Порядок действий:
    30 – 20 = 10
    35 – 10 = 25
    100 – 25 = 75
    75 · 2 = 150

    Какие действия нужно выполнять. Порядок выполнения действий в выражениях без скобок и со скобками. Вставь пропущенное число

    Числовые,буквенные выражения и выражения с переменными в своей записи могут содержать знаки различных арифметических действий. При преобразовании выражений и вычислении значений выражений действия выполняются в определенной очередности, иными словами, нужно соблюдать порядок выполнения действий .

    В этой статье мы разберемся, какие действия следует выполнять сначала, а какие следом за ними. Начнем с самых простых случаев, когда выражение содержит лишь числа или переменные, соединенные знаками плюс, минус, умножить и разделить. Дальше разъясним, какого порядка выполнения действий следует придерживаться в выражениях со скобками. Наконец, рассмотрим, в какой последовательности выполняются действия в выражениях, содержащих степени, корни и другие функции.

    Навигация по странице.

    В школе дается следующее правило, определяющее порядок выполнения действий в выражениях без скобок :

  • действия выполняются по порядку слева направо,
  • причем сначала выполняется умножение и деление, а затем – сложение и вычитание.
  • Озвученное правило воспринимается достаточно естественно. Выполнение действий по порядку слева направо объясняется тем, что у нас принято вести записи слева направо. А то, что умножение и деление выполняется перед сложением и вычитанием объясняется смыслом, который в себе несут эти действия.

    Рассмотрим несколько примеров применения этого правила. Для примеров будем брать простейшие числовые выражения, чтобы не отвлекаться на вычисления, а сосредоточиться именно на порядке выполнения действий.

    Выполните действия 7−3+6 .

    Исходное выражение не содержит скобок, а также оно не содержит умножения и деления. Поэтому нам следует выполнить все действия по порядку слева направо, то есть, сначала мы от 7 отнимаем 3 , получаем 4 , после чего к полученной разности 4 прибавляем 6 , получаем 10 .

    Кратко решение можно записать так: 7−3+6=4+6=10 .

    Укажите порядок выполнения действий в выражении 6:2·8:3 .

    Чтобы ответить на вопрос задачи, обратимся к правилу, указывающему порядок выполнения действий в выражениях без скобок. В исходном выражении содержатся лишь действия умножения и деления, а согласно правилу, их нужно выполнять по порядку слева направо.

    сначала 6 делим на 2 , это частное умножаем на 8 , наконец, полученный результат делим на 3.

    Вычислите значение выражения 17−5·6:3−2+4:2 .

    Сначала определим, в каком порядке следует выполнять действия в исходном выражении. Оно содержит и умножение с делением, и сложение с вычитанием. Сначала слева направо нужно выполнить умножение и деление. Так 5 умножаем на 6 , получаем 30 , это число делим на 3 , получаем 10 . Теперь 4 делим на 2 , получаем 2 . Подставляем в исходное выражение вместо 5·6:3 найденное значение 10 , а вместо 4:2 — значение 2 , имеем 17−5·6:3−2+4:2=17−10−2+2 .

    В полученном выражении уже нет умножения и деления, поэтому остается по порядку слева направо выполнить оставшиеся действия: 17−10−2+2=7−2+2=5+2=7 .

    На первых порах, чтобы не перепутать порядок выполнения действий при вычислении значения выражения, удобно над знаками действий расставить цифры, соответствующие порядку их выполнения. Для предыдущего примера это выглядело бы так: .

    Этого же порядка выполнения действий – сначала умножение и деление, затем сложение и вычитание — следует придерживаться и при работе с буквенными выражениями.

    Действия первой и второй ступени

    В некоторых учебниках по математике встречается разделение арифметических действий на действия первой и второй ступени. Разберемся с этим.

    Действиями первой ступени называют сложение и вычитание, а умножение и деление называют действиями второй ступени .

    В этих терминах правило из предыдущего пункта, определяющее порядок выполнения действий, запишется так: если выражение не содержит скобок, то по порядку слева направо сначала выполняются действия второй ступени (умножение и деление), затем – действия первой ступени (сложение и вычитание).

    Порядок выполнения арифметических действий в выражениях со скобками

    Выражения часто содержат скобки, указывающие порядок выполнения действий. В этом случае правило, задающее порядок выполнения действий в выражениях со скобками , формулируется так: сначала выполняются действия в скобках, при этом также по порядку слева направо выполняется умножение и деление, затем – сложение и вычитание.

    Итак, выражения в скобках рассматриваются как составные части исходного выражения, и в них сохраняется уже известный нам порядок выполнения действий. Рассмотрим решения примеров для большей ясности.

    Выполните указанные действия 5+(7−2·3)·(6−4):2 .

    Выражение содержит скобки, поэтому сначала выполним действия в выражениях, заключенных в эти скобки. Начнем с выражения 7−2·3 . В нем нужно сначала выполнить умножение, и только потом вычитание, имеем 7−2·3=7−6=1 . Переходим ко второму выражению в скобках 6−4 . Здесь лишь одно действие – вычитание, выполняем его 6−4=2 .

    Подставляем полученные значения в исходное выражение: 5+(7−2·3)·(6−4):2=5+1·2:2 . В полученном выражении сначала выполняем слева направо умножение и деление, затем – вычитание, получаем 5+1·2:2=5+2:2=5+1=6 . На этом все действия выполнены, мы придерживались такого порядка их выполнения: 5+(7−2·3)·(6−4):2 .

    Запишем краткое решение: 5+(7−2·3)·(6−4):2=5+1·2:2=5+1=6 .

    Бывает, что выражение содержит скобки в скобках. Этого бояться не стоит, нужно лишь последовательно применять озвученное правило выполнения действий в выражениях со скобками. Покажем решение примера.

    Выполните действия в выражении 4+(3+1+4·(2+3)) .

    Это выражение со скобками, это означает, что выполнение действий нужно начинать с выражения в скобках, то есть, с 3+1+4·(2+3) . Это выражение также содержит скобки, поэтому нужно сначала выполнить действия в них. Сделаем это: 2+3=5 . Подставив найденное значение, получаем 3+1+4·5 . В этом выражении сначала выполняем умножение, затем – сложение, имеем 3+1+4·5=3+1+20=24 . Исходное значение, после подстановки этого значения, принимает вид 4+24 , и остается лишь закончить выполнение действий: 4+24=28 .

    Вообще, когда в выражении присутствуют скобки в скобках, то часто бывает удобно выполнение действий начинать с внутренних скобок и продвигаться к внешним.

    Например, пусть нам нужно выполнить действия в выражении (4+(4+(4−6:2))−1)−1 . Сначала выполняем действия во внутренних скобках, так как 4−6:2=4−3=1 , то после этого исходное выражение примет вид (4+(4+1)−1)−1 . Опять выполняем действие во внутренних скобках, так как 4+1=5 , то приходим к следующему выражению (4+5−1)−1 . Опять выполняем действия в скобках: 4+5−1=8 , при этом приходим к разности 8−1 , которая равна 7 .

    Порядок выполнения действий в выражениях с корнями, степенями, логарифмами и другими функциями

    Если в выражение входят степени, корни, логарифмы, синус, косинус, тангенс и котангенс, а также другие функции, то их значения вычисляются до выполнения остальных действий, при этом также учитываются правила из предыдущих пунктов, задающие порядок выполнения действий. Иными словами, перечисленные вещи, грубо говоря, можно считать заключенными в скобки, а мы знаем, что сначала выполняются действия в скобках.

    Рассмотрим решения примеров.

    Выполните действия в выражении (3+1)·2+6 2:3−7 .

    В этом выражении содержится степень 6 2 , ее значение нужно вычислить до выполнения остальных действий. Итак, выполняем возведение в степень: 6 2 =36 . Подставляем это значение в исходное выражение, оно примет вид (3+1)·2+36:3−7 .

    Дальше все понятно: выполняем действия в скобках, после чего остается выражение без скобок, в котором по порядку слева направо сначала выполняем умножение и деление, а затем – сложение и вычитание. Имеем (3+1)·2+36:3−7=4·2+36:3−7= 8+12−7=13 .

    Другие, в том числе и более сложные примеры выполнения действий в выражениях с корнями, степенями и т.п., Вы можете посмотреть в статье вычисление значений выражений.

    cleverstudents.ru

    Онлайн игры,тренажеры,презентации,уроки,энциклопедии,статьи

    Post navigation

    Примеры со скобками, урок с тренажерами.

    Мы рассмотрим в этой статье три варианта примеров:

    1. Примеры со скобками (действия сложения и вычитания)

    2. Примеры со скобками (сложение, вычитание, умножение, деление)

    3. Примеры, в которых много действий

    1 Примеры со скобками (действия сложения и вычитания)

    Рассмотрим три примера. В каждом из них порядок действий обозначен цифрами красного цвета:

    Мы видим, что порядок действий в каждом примере будет разный, хотя числа и знаки одинаковые. Это происходит потому, что во втором и третьем примере есть скобки.

  • Если в примере нет скобок , мы выполняем все действия по порядку, слева направо.
  • Если в примере есть скобки , то сначала мы выполняем действия в скобках, и лишь потом все остальные действия, начиная слева направо.
  • *Это правило для примеров без умножения и деления. Правила для примеров со скобками, включающих действия умножения и деления мы рассмотрим во второй части этой статьи.

    Чтобы не запутаться в примере со скобками, можно превратить его в обычный пример, без скобок. Для этого результат, полученный в скобках, записываем над скобками, далее переписываем весь пример, записывая вместо скобок этот результат, и далее выполняем все действия по порядку, слева направо:

    В несложных примерах можно все эти операции производить в уме. Главное — сначала выполнить действие в скобках и запомнить результат, а затем считать по порядку, слева направо.

    А теперь — тренажеры!

    1) Примеры со скобками в пределах до 20.

    Онлайн тренажер.

    2) Примеры со скобками в пределах до 100. Онлайн тренажер.

    3) Примеры со скобками. Тренажер №2

    4) Вставь пропущенное число — примеры со скобками. Тренажер

    2 Примеры со скобками (сложение, вычитание, умножение, деление)

    Теперь рассмотрим примеры, в которых кроме сложения и вычитания есть умножение и деление.

    Сначала рассмотрим примеры без скобок:

  • Если в примере нет скобок , сначала выполняем действия умножения и деления по порядку, слева направо. Затем — действия сложения и вычитания по порядку, слева направо.
  • Если в примере есть скобки , то сначала мы выполняем действия в скобках, затем умножение и деление, и затем — сложение и вычитание начиная слева направо.
  • Есть одна хитрость, как не запутаться при решении примеров на порядок действий. Если нет скобок, то выполняем действия умножения и деления, далее переписываем пример, записывая вместо этих действий полученные результаты. Затем выполняем сложение и вычитание по порядку:

    Если в примере есть скобки, то сначала нужно избавиться от скобок: переписать пример, записывая вместо скобок полученный в них результат. Затем нужно выделить мысленно части примера, разделенные знаками «+» и «-«, и посчитать каждую часть отдельно. Затем выполнить сложение и вычитание по порядку:

    3 Примеры, в которых много действий

    Если в примере много действий, то удобнее будет не расставлять порядок действий во всем примере, а выделить блоки, и решить каждый блок отдельно. Для этого находим свободные знаки «+» и «–» (свободные — значит не в скобках, на рисунке показаны стрелочками).

    Эти знаки и будут делить наш пример на блоки:

    Выполняя действия в каждом блоке не забываем про порядок действий, приведенный выше в статье. Решив каждый блок, выполняем действия сложения и вычитания по порядку.

    А теперь закрепляем решение примеров на порядок действий на тренажерах!

    1.

    Примеры со скобками в пределах чисел до 100, действия сложения, вычитания, умножения и деления. Онлайн тренажер.

    2. Тренажер по математике 2 — 3 класс «Расставь порядок действий (буквенные выражения).»

    3. Порядок действий (расставляем порядок и решаем примеры)

    Порядок действий в математике 4 класс

    Начальная школа подходит к концу, скоро ребёнок шагнёт в углубленный мир математики. Но уже в этот период школьник сталкивается с трудностями науки. Выполняя простое задание, ребёнок путается, теряется, что в результате приводит к отрицательной отметке за выполненную работу. Чтобы избежать подобных неприятностей, нужно при решении примеров, уметь ориентироваться в порядке, по которому нужно решать пример. Не верно распределив действия, ребёнок не правильно выполняет задание. В статье раскрываются основные правила решения примеров, содержащих в себе весь спектр математических вычислений, включая скобки. Порядок действий в математике 4 класс правила и примеры.

    Перед выполнением задания попросите своё чадо пронумеровать действия, которые он собирается выполнить. Если возникли затруднения – помогите.

    Некоторые правила, которые необходимо соблюдать при решении примеров без скобок:

    Если в задании необходимо выполнить ряд действий, нужно сначала выполнить деление или умножение, затем сложение. Все действия выполняются по ходу письма. В противном случае, результат решения будет не верным.

    Если в примере требуется выполнить сложение и вычитание, выполняем по порядку, слева направо.

    27-5+15=37 (при решении примера руководствуемся правилом. Сначала выполняем вычитание, затем – сложение).

    Научите ребёнка всегда планировать и нумеровать выполняемые действия.

    Ответы на каждое решённое действие записываются над примером. Так ребёнку гораздо легче будет ориентироваться в действиях.

    Рассмотрим ещё один вариант, где необходимо распределить действия по порядку:

    Как видим, при решении соблюдено правило, сначала ищем произведение, после — разность.

    Это простые примеры, при решении которых, необходима внимательность. Многие дети впадают в ступор при виде задания, в котором присутствует не только умножение и деление, но и скобки. У школьника, не знающего порядок выполнения действий, возникают вопросы, которые мешают выполнить задание.

    Как говорилось в правиле, сначала найдём произведение или частное, а потом всё остальное. Но тут же есть скобки! Как поступить в этом случае?

    Решение примеров со скобками

    Разберём конкретный пример:

  • При выполнении данного задания, сначала найдём значение выражения, заключённого в скобки.
  • Начать следует с умножения, далее – сложение.
  • После того, как выражение в скобках решено, приступаем к действиям вне их.
  • По правилам порядка действий, следующим шагом будет умножение.
  • Завершающим этапом станет вычитание.
  • Как видим на наглядном примере, все действия пронумерованы. Для закрепления темы предложите ребёнку решить самостоятельно несколько примеров:

    Порядок, по которому следует вычислять значение выражения уже расставлен. Ребёнку останется только выполнить непосредственно решение.

    Усложним задачу. Пусть ребёнок найдёт значение выражений самостоятельно.

    7*3-5*4+(20-19) 14+2*3-(13-9)
    17+2*5+(28-2) 5*3+15-(2-1*2)
    24-3*2-(56-4*3) 14+12-3*(21-7)

    Приучите ребёнка решать все задания в черновом варианте. В таком случае, у школьника будет возможность исправить не верное решение или помарки. В рабочей тетради исправления не допустимы. Выполняя самостоятельно задания, дети видят свои ошибки.

    Родители, в свою очередь, должны обратить внимание на ошибки, помочь ребёнку разобраться и исправить их. Не стоит нагружать мозг школьника большими объёмами заданий. Такими действиями вы отобьёте стремление ребёнка к знаниям. Во всём должно быть чувство меры.

    Делайте перерыв. Ребёнок должен отвлекаться и отдыхать от занятий. Главное помнить, что не все обладают математическим складом ума. Может из вашего ребёнка вырастет знаменитый философ.

    detskoerazvitie. info

    Урок по математике 2 класс Порядок действий в выражениях со скобками.

    Успейте воспользоваться скидками до 50% на курсы «Инфоурок»

    Цель: 1.

    2.

    3. Закрепить знание таблицы умножения и деления на 2 – 6, понятия делителя и

    4. Учить работать в парах с целью развития коммуникативных качеств.

    Оборудование * : + — (), геометрический материал.

    Раз, два – выше голова.

    Три, четыре – руки шире.

    Пять, шесть – всем присесть.

    Семь, восемь – лень отбросим.

    Но сначала придется узнать его название. Для этого нужно выполнить несколько заданий:

    6 + 6 + 6 … 6 * 4 6 * 4 + 6… 6 * 5 – 6 14 дм 5 см… 4 дм 5 см

    Пока мы вспоминали о порядке действий в выражениях, с замком происходили чудеса. Мы были только что у ворот, а теперь попали в коридор. Смотрите, дверь. А на ней замок. Откроем?

    1. Из числа 20 вычесть частное чисел 8 и 2.

    2. Разность чисел 20 и 8 разделить на 2.

    — Чем отличаются результаты?

    — Кто сможет назвать тему нашего урока?

    (на массажных ковриках)

    По дорожке, по дорожке

    Скачем мы на правой ножке,

    Скачем мы на левой ножке.

    По тропинке побежим,

    Наше предположение было полностью правильно7

    Где выполняются действия сначала, если в выражении есть скобки?

    Смотрите перед нами «живые примеры». Давайте «оживим» их.

    * : + — ().

    m – c * (a + d) + x

    k: b + (a – c) * t

    6. Работа в парах.

    Для их решения вам понадобиться геометрический материал.

    Учащиеся выполняют задания в парах. После выполнения проверка работы пар у доски.

    Что нового вы узнали?

    8. Домашнее задание.

    Тема: Порядок действий в выражениях со скобками.

    Цель: 1. Вывести правило порядка действий в выражениях со скобками, содержащих все

    4 арифметических действия,

    2. Формировать способность к практическому применению правила,

    4.Учить работать в парах с целью развития коммуникативных качеств.

    Оборудование : учебник, тетради, карточки со знаками действий * : + — (), геометрический материал.

    1 .Физминутка.

    Девять, десять – тихо сесть.

    2. Актуализация опорных знаний.

    Сегодня мы с вами отправляемся в очередное путешествие по стране Знаний городу математика. Нам предстоит посетить один дворец. Что-то я забыла его название. Но не будем расстраиваться, вы сами сможете мне подсказать его название. Пока я переживала, мы подошли к воротам дворца. Войдем?

    1. Сравните выражения:

    2. Расшифруй слово.

    3. Постановка проблемы. Открытие нового.

    Так как же называется дворец?

    А когда в математике мы говорим о порядке?

    Что вы уже знаете о порядке выполнения действий в выражениях?

    — Интересно, нам предлагают записать и решить выражения (учитель читает выражения, учащиеся записывают их и решают).

    20 – 8: 2

    (20 – 8) : 2

    Молодцы. А что интересного в этих выражениях?

    Посмотрите на выражения и их результаты.

    — Что общего в записи выражений?

    — Как вы думаете, почему получились разные результаты, ведь числа были одинаковые?

    Кто рискнет сформулировать правило выполнения действий в выражениях со скобками?

    Правильность этого ответа мы сможем проверить в другой комнате. Отправляемся туда.

    4. Физминутка.

    И по этой же дорожке

    До горы мы добежим.

    Стоп. Немножко отдохнем

    И опять пешком пойдем.

    5. Первичное закрепление изученного.

    Вот мы и пришли.

    Нам нужно решить еще два выражения, чтобы проверить правильность нашего предположения.

    6 * (33 – 25) 54: (6 + 3) 25 – 5 * (9 – 5) : 2

    Для проверки правильности предположения откроем учебники на стр. 33 и прочитаем правило.

    Как нужно выполнять действия после решения в скобках?

    На доске написаны буквенные выражения и лежат карточки со знаками действий * : + — (). Дети выходят к доске по одному, берут карточку с тем действием, которое нужно сделать сначала, потом выходит второй ученик и берет карточку со вторым действием и т. д.

    а + (а –в)

    а * (в +с) : d t

    m c * ( a + d ) + x

    k : b + ( a c ) * t

    (a – b) : t + d

    6. Работа в парах.

    Знание порядка действий необходимо не только для решения примеров, но и при решении задач мы тоже сталкиваемся с этим правилом. Сейчас вы в этом убедитесь работая в парах. Вам нужно будет решить задачи из № 3 стр. 33.

  • Особенности бухгалтерского учета субсидий Государство стремится поддержать малое и среднее предпринимательство. Такая поддержка наиболее часто выражается в форме предоставления субсидий – безвозмездных выплат из […]
  • Жалоба на педиатра Жалоба на педиатра — официальный документ, устанавливающий требования пациента и описывающий суть возникновения таких требований. Согласно статье 4 Федерального закона «О порядке рассмотрения […]
  • Ходатайство об уменьшении размера исковых требований Один из видов уточнения иска — ходатайство об уменьшении размера исковых требований. Когда истец неправильно определил цену иска. Или ответчик частично исполнил […]
  • Черный рынок доллара в Киеве Валютный аукцион по покупке доллара в Киеве Внимание: администрация не несёт ответственности за содержание объявлений на валютном аукционе.
    Правила публикации объявлений на валютном […]
  • Примеры со скобками, урок с тренажерами. — Kid-mama

    Мы рассмотрим в этой статье три варианта примеров:

    1. Примеры со скобками (действия сложения и вычитания)

    2. Примеры со скобками (сложение, вычитание, умножение, деление)

    3. Примеры, в которых много действий

    1 Примеры со скобками (действия сложения и вычитания)

    Рассмотрим три примера. В каждом из них порядок действий обозначен цифрами красного цвета:

    Мы видим, что порядок действий  в каждом примере будет разный, хотя числа и знаки одинаковые. Это происходит потому, что во втором и третьем примере есть скобки.

    Запомните правило:

    • Если в примере нет скобок, мы выполняем все действия по порядку, слева направо.
    • Если  в примере есть скобки, то сначала мы выполняем действия в скобках, и лишь потом все остальные действия, начиная слева направо.

    *Это правило для примеров без умножения и деления. Правила для примеров со скобками, включающих действия умножения и деления мы рассмотрим во второй части этой статьи. 

    Чтобы не запутаться в примере со скобками, можно превратить его в обычный пример, без скобок. Для этого результат, полученный в скобках, записываем над скобками, далее переписываем весь пример, записывая вместо скобок этот результат, и далее выполняем  все действия по порядку, слева направо:

    В несложных примерах можно все эти операции производить в уме. Главное — сначала выполнить действие в скобках и запомнить результат, а затем считать по порядку, слева направо.

    А теперь — тренажеры!

    1) Примеры со скобками в пределах до 20. Онлайн тренажер.

    Перейти на страницу  с тренажером

    2) Примеры со скобками в пределах до 100. Онлайн тренажер.

    Перейти на страницу  с тренажером

    3) Примеры со скобками.

    Тренажер №2

    Перейти на страницу  с тренажером

    4) Вставь пропущенное число — примеры со скобками. Тренажер

    Перейти на страницу  с тренажером

    2 Примеры со скобками (сложение, вычитание, умножение, деление)

    Теперь рассмотрим примеры, в которых кроме сложения и вычитания есть умножение и деление.

    Сначала рассмотрим примеры без скобок:

    Запомните правило:

    • Если в примере нет скобок, сначала выполняем действия умножения и деления по порядку, слева направо. Затем — действия сложения и вычитания по порядку, слева направо.
    • Если  в примере есть скобки, то сначала мы выполняем действия в скобках, затем умножение и деление, и затем — сложение и вычитание начиная слева направо.

    Есть одна хитрость, как не запутаться при решении примеров на порядок действий. Если нет скобок, то выполняем действия умножения и деления, далее переписываем пример, записывая вместо этих действий  полученные результаты.   Затем выполняем сложение и вычитание по порядку:

    Если в примере есть скобки, то сначала нужно избавиться от скобок: переписать пример, записывая вместо скобок полученный в них результат. Затем нужно выделить мысленно части примера, разделенные знаками «+» и «-«, и посчитать каждую часть отдельно. Затем выполнить сложение и вычитание по порядку:

    3 Примеры, в которых много действий

    Если в примере много действий, то удобнее будет не расставлять порядок действий во всем примере, а выделить блоки, и решить каждый блок отдельно.  Для этого находим свободные знаки «+» и «–» (свободные — значит не в скобках, на рисунке показаны стрелочками).

    Эти знаки и будут делить наш пример на блоки:

     Выполняя действия в каждом блоке не забываем про порядок действий, приведенный выше в статье. Решив каждый блок, выполняем действия сложения и вычитания по порядку.

    А теперь закрепляем решение примеров на порядок действий на тренажерах!

    1. Примеры со скобками в пределах чисел до 100, действия сложения, вычитания, умножения и деления.

    Онлайн тренажер.

    Перейти на страницу  с тренажером

    Перейти на страницу  с тренажером

    3. Порядок действий (расставляем порядок и решаем примеры)

    Перейти на страницу  с тренажером

    3 класс. Моро. Учебник №1. Ответы к стр. 24

    Числа от 1 до 100

    Умножение и деление
    Порядок выполнения действий
    Ответы к стр. 24

    Узнаем, в каком порядке выполняются действия в числовых выражениях.

    1. Сравни выражения каждой пары: какие действия в них выполняются? В каком порядке выполняются эти действия и почему?
    38 — 10 + 6 = 28 + 6 = 34        24 : 3 • 2 = 8 • 2 = 16
    38 — (10 + 6) = 28 + 6 = 34     24 : (3 • 2) = 8 • 2 = 16
    Прежде чем приступить к вычислениям, надо рассмотреть выражение: выяснить, есть ли в нём скобки, какие действия в нём имеются.
    1) Если в выражение без скобок входят только сложение и вычитание или только умножение и деление, то действия выполняются в том порядке, в каком они записаны: слева направо.
    Если в выражение без скобок входят не только сложение и вычитание, но и умножение или деление или оба этих действия, то сначала выполняют по порядку (слева направо) умножение и деление, а затем сложение и вычитание.
    3) Если в выражении есть скобки, то сначала выполняют действия, записанные в скобках, по правилам пунктов 1 и 2.

    Действия в числовых выражениях выполняют в следующем порядке:
    1) действия, записанные в скобках;
    2) умножение и деление;
    3) сложение и вычитание.

    В первой паре выполняется сложение и вычитание, во второй — деление и умножение. В первой случае сначала выполняется вычитание, а затем деление. Но при наличии скобок сначала выполняются действия в скобках — сложение, а затем вычитание.

    Во втором случае сначала выполняется деление, а затем умножение. Но при наличии скобок сначала выполняются действия в скобках — умножение, а затем деление.

    2. Объясни, как надо выполнять действия.
          3     2       1
    30 + 6 • (13 — 9) = 30 + 6 • 4 = 30 + 24 = 54
          1     4     2      5        3
    18 : 2 — 2 • 3 + 12 : 3 = 9 — 6 + 4 = 7

    В первом выражении сначала выполняется действие в скобках — вычитание, затем умножение, а потом сложение.

    Во втором выражении сначала выполняются действия деления и умножения, а затем действия вычитания и сложения — слева направо.

    ГДЗ по математике. Учебник. 3 класс. Часть 1. Моро М. И., Бантова М. А., Бельтюкова М. А., Волкова С. И., Степанова С. В.

    Математика. 3 класс

    3 класс. Моро. Учебник №1. Ответы к стр. 24

    3.9 (78.46%) от 13 голосующих

    Сложение, вычитание, умножение и деление. ереместительное, сочетательное свойства. Примеры решение задач.

    Арифметические операции

    Сложение:

    Умножение:

    Вычитание:

     

     Деление:

     Переместительное свойство

    Это свойство относится только к двум операциям: сложение и умножение, так как только в этих операциях каждое из слагаемых или множителей имеет одинаковое значение.

    Cочетательное свойство.

    Следующее свойство – сочетательное. Это свойство рассматривается для сложения и умножения.

     

    Переместительное и сочетательное свойства для сложения и умножения позволяют объединять слагаемые и множители в группы, менять их местами. Эти свойства позволяют считать быстрее и без ошибок.

    Распределительные свойства

    Следующие свойства раcпределительные. Они показывают, как можно вычислить выражение, если в нем используются операция умножение вместе со сложением или вычитанием (распределяют порядок вычисления):

     

    Противоположный элемент

     

    Нейтральный элемент – 0.

    Ноль — это нейтральный элемент относительно сложения целых чисел:

    Также обрати внимание на порядок  действий, если скобки не расставлены. Итак, у нас есть 4 операции, они выполняются в следующем порядке:

    1.  Умножение и деление – в порядке следования слева направо;
    2.  Сложение и вычитание – в порядке следования слева направо.
    3. При наличии скобок сначала выполняются действия в скобках в указанном выше порядке, а затем все остальные действия вне скобок опять же с соблюдением указанного выше порядка.

    Задача 1. Вычислить  \(-55+(-7)+18+7.\)

    Решение.

    1. Воспользуемся переместительным свойством для удобства вычисления: \(-7+7-55+18\)

     

    1. \(-7\) и \(7\) противоположные элементы, итого: \(-55+18=-37\)

    Ответ:\(-37\)

    Задача 1. Вычислить   \((-7+9)+7*2-56\).

    1. Первое действие выполняем в скобках и умножение: \(2+ 7*2\)
    2. выполняем умножение, затем сложение и вычитание: \(2+14-56=16-56=-40.\)

    Ответ:\(-40.\)

    Запишись на бесплатный пробный урок тут и разберись с тем, что тебе непонятно.

     

     

     

     

     

    Больше уроков и заданий по математике вместе с преподавателями нашей онлайн-школы «Альфа». Запишитесь на пробное занятие уже сейчас!

    Запишитесь на бесплатное тестирование знаний!

    Что сначала делают умножение или деление

    Когда мы работаем с различными выражениями, включающими в себя цифры, буквы и переменные, нам приходится выполнять большое количество арифметических действий. Когда мы делаем преобразование или вычисляем значение, очень важно соблюдать правильную очередность этих действий. Иначе говоря, арифметические действия имеют свой особый порядок выполнения.

    В этой статье мы расскажем, какие действия надо делать в первую очередь, а какие после. Для начала разберем несколько простых выражений, в которых есть только переменные или числовые значения, а также знаки деления, умножения, вычитания и сложения. Потом возьмем примеры со скобками и рассмотрим, в каком порядке следует вычислять их. В третьей части мы приведем нужный порядок преобразований и вычислений в тех примерах, которые включают в себя знаки корней, степеней и других функций.

    Порядок вычисления простых выражений

    В случае выражений без скобок порядок действий определяется однозначно:

    1. Все действия выполняются слева направо.
    2. В первую очередь мы выполняем деление и умножение, во вторую – вычитание и сложение.

    Смысл этих правил легко уяснить. Традиционный порядок записи слева направо определяет основную последовательность вычислений, а необходимость сначала умножить или разделить объясняется самой сутью этих операций.

    Возьмем для наглядности несколько задач. Мы использовали только самые простые числовые выражения, чтобы все вычисления можно было провести в уме. Так можно быстрее запомнить нужный порядок и быстро проверить результаты.

    Условие: вычислите, сколько будет 7 − 3 + 6 .

    Решение

    В нашем выражении скобок нет, умножение и деление также отсутствуют, поэтому выполняем все действия в указанном порядке. Сначала вычитаем три из семи, затем прибавляем к остатку шесть и в итоге получаем десять. Вот запись всего решения:

    7 − 3 + 6 = 4 + 6 = 10

    Ответ: 7 − 3 + 6 = 10 .

    Условие: в каком порядке нужно выполнять вычисления в выражении 6 : 2 · 8 : 3 ?

    Решение

    Чтобы дать ответ на этот вопрос, перечитаем правило для выражений без скобок, сформулированное нами до этого. У нас здесь есть только умножение и деление, значит, мы сохраняем записанный порядок вычислений и считаем последовательно слева направо.

    Ответ: сначала выполняем деление шести на два, результат умножаем на восемь и получившееся в итоге число делим на три.

    Условие: подсчитайте, сколько будет 17 − 5 · 6 : 3 − 2 + 4 : 2 .

    Решение

    Сначала определим верный порядок действий, поскольку у нас здесь есть все основные виды арифметических операций – сложение, вычитание, умножение, деление. Первым делом нам надо разделить и умножить. Эти действия не имеют приоритета друг перед другом, поэтому выполняем их в написанном порядке справа налево. То есть 5 надо умножить на 6 и получить 30 , потом 30 разделить на 3 и получить 10 . После этого делим 4 на 2 , это 2 . Подставим найденные значения в исходное выражение:

    17 − 5 · 6 : 3 − 2 + 4 : 2 = 17 − 10 − 2 + 2

    Здесь уже нет ни деления, ни умножения, поэтому делаем оставшиеся вычисления по порядку и получаем ответ:

    17 − 10 − 2 + 2 = 7 − 2 + 2 = 5 + 2 = 7

    Ответ: 17 − 5 · 6 : 3 − 2 + 4 : 2 = 7 .

    Пока порядок выполнения действий не заучен твердо, можно ставить над знаками арифметических действий цифры, означающие порядок вычисления. Например, для задачи выше мы могли бы записать так:

    .

    Если у нас есть буквенные выражения, то с ними мы поступаем точно так же: сначала умножаем и делим, затем складываем и вычитаем.

    Что такое действия первой и второй ступени

    Иногда в справочниках все арифметические действия делят на действия первой и второй ступени. Сформулируем нужное определение.

    К действиям первой ступени относятся вычитание и сложение, второй – умножение и деление.

    Зная эти названия, мы можем записать данное ранее правило относительно порядка действий так:

    В выражении, в котором нет скобок, сначала надо выполнить действия второй ступени в направлении слева направо, затем действия первой ступени (в том же направлении).

    Порядок вычислений в выражениях со скобками

    Скобки сами по себе являются знаком, который сообщает нам нужный порядок выполнения действий. В таком случае нужное правило можно записать так:

    Если в выражении есть скобки, то первым делом выполняется действие в них, после чего мы умножаем и делим, а затем складываем и вычитаем по направлению слева направо.

    Что касается самого выражения в скобках, его можно рассматривать в качестве составной части основного выражения. При подсчете значения выражения в скобках мы сохраняем все тот же известный нам порядок действий. Проиллюстрируем нашу мысль примером.

    Условие: вычислите, сколько будет 5 + ( 7 − 2 · 3 ) · ( 6 − 4 ) : 2 .

    Решение

    В данном выражении есть скобки, поэтому начнем с них. Первым делом вычислим, сколько будет 7 − 2 · 3 . Здесь нам надо умножить 2 на 3 и вычесть результат из 7 :

    7 − 2 · 3 = 7 − 6 = 1

    Считаем результат во вторых скобках. Там у нас всего одно действие: 6 − 4 = 2 .

    Теперь нам нужно подставить получившиеся значения в первоначальное выражение:

    5 + ( 7 − 2 · 3 ) · ( 6 − 4 ) : 2 = 5 + 1 · 2 : 2

    Начнем с умножения и деления, потом выполним вычитание и получим:

    5 + 1 · 2 : 2 = 5 + 2 : 2 = 5 + 1 = 6

    На этом вычисления можно закончить.

    Ответ: 5 + ( 7 − 2 · 3 ) · ( 6 − 4 ) : 2 = 6 .

    Не пугайтесь, если в условии у нас содержится выражение, в котором одни скобки заключают в себе другие. Нам надо только применять правило выше последовательно по отношению ко всем выражениям в скобках. Возьмем такую задачу.

    Условие: вычислите, сколько будет 4 + ( 3 + 1 + 4 · ( 2 + 3 ) ) .

    Решение

    У нас есть скобки в скобках. Начинаем с 3 + 1 + 4 · ( 2 + 3 ) , а именно с 2 + 3 . Это будет 5 . Значение надо будет подставить в выражение и подсчитать, что 3 + 1 + 4 · 5 . Мы помним, что сначала надо умножить, а потом сложить: 3 + 1 + 4 · 5 = 3 + 1 + 20 = 24 . Подставив найденные значения в исходное выражение, вычислим ответ: 4 + 24 = 28 .

    Ответ: 4 + ( 3 + 1 + 4 · ( 2 + 3 ) ) = 28 .

    Иначе говоря, при вычислении значения выражения, включающего скобки в скобках, мы начинаем с внутренних скобок и продвигаемся к внешним.

    Допустим, нам надо найти, сколько будет ( 4 + ( 4 + ( 4 − 6 : 2 ) ) − 1 ) − 1 . Начинаем с выражения во внутренних скобках. Поскольку 4 − 6 : 2 = 4 − 3 = 1 , исходное выражение можно записать как ( 4 + ( 4 + 1 ) − 1 ) − 1 . Снова обращаемся к внутренним скобкам: 4 + 1 = 5 . Мы пришли к выражению ( 4 + 5 − 1 ) − 1 . Считаем 4 + 5 − 1 = 8 и в итоге получаем разность 8 – 1 , результатом которой будет 7 .

    Порядок вычисления в выражениях со степенями, корнями, логарифмами и иными функциями

    Если у нас в условии стоит выражение со степенью, корнем, логарифмом или тригонометрической функцией (синусом, косинусом, тангенсом и котангенсом) или иными функциями, то первым делом мы вычисляем значение функции. После этого мы действуем по правилам, указанным в предыдущих пунктах. Иначе говоря, функции по степени важности приравниваются к выражению, заключенному в скобки.

    Разберем пример такого вычисления.

    Условие: найдите, сколько будет ( 3 + 1 ) · 2 + 6 2 : 3 − 7 .

    Решение

    У нас есть выражение со степенью, значение которого надо найти в первую очередь. Считаем: 6 2 = 36 . Теперь подставим результат в выражение, после чего оно примет вид ( 3 + 1 ) · 2 + 36 : 3 − 7 .

    Дальше действуем по знакомому алгоритму: считаем, сколько у нас получится в скобках, потом в оставшемся выражении выполняем умножение и деление, а следом – сложение и вычитание.

    ( 3 + 1 ) · 2 + 36 : 3 − 7 = 4 · 2 + 36 : 3 − 7 = 8 + 12 − 7 = 13

    Ответ: ( 3 + 1 ) · 2 + 6 2 : 3 − 7 = 13 .

    В отдельной статье, посвященной вычислению значений выражений, мы приводим и другие, более сложные примеры подсчетов в случае выражений с корнями, степенью и др. Рекомендуем вам с ней ознакомиться.

    На данном уроке подробно рассмотрен порядок выполнения арифметических действий в выражениях без скобок и со скобками. Учащимся предоставляется возможность в ходе выполнения заданий определить, зависит ли значение выражений от порядка выполнения арифметических действий, узнать отличается ли порядок арифметических действий в выражениях без скобок и со скобками, потренироваться в применении изученного правила, найти и исправить ошибки, допущенные при определении порядка действий.

    Наблюдение за изменением значения выражения от порядка выполнения арифметических действий

    В жизни мы постоянно выполняем какие-либо действия: гуляем, учимся, читаем, пишем, считаем, улыбаемся, ссоримся и миримся. Эти действия мы выполняем в разном порядке. Иногда их можно поменять местами, а иногда нет. Например, собираясь утром в школу, можно сначала сделать зарядку, затем заправить постель, а можно наоборот. Но нельзя сначала уйти в школу, а потом надеть одежду.

    А в математике обязательно ли выполнять арифметические действия в определенном порядке?

    Сравним выражения:
    8-3+4 и 8-3+4

    Видим, что оба выражения совершенно одинаковы.

    Выполним действия в одном выражения слева направо, а в другом справа налево. Числами можно проставить порядок выполнения действий (рис. 1).

    Рис. 1. Порядок действий

    В первом выражении мы сначала выполним действие вычитания, а затем к результату прибавим число 4.

    Во втором выражении сначала найдем значение суммы, а потом из 8 вычтем полученный результат 7.

    Видим, что значения выражений получаются разные.

    Сделаем вывод: порядок выполнения арифметических действий менять нельзя.

    Порядок выполнения арифметических действий в выражениях без скобок

    Узнаем правило выполнения арифметических действий в выражениях без скобок.

    Если в выражение без скобок входят только сложение и вычитание или только умножение и деление, то действия выполняют в том порядке, в каком они написаны.

    В этом выражении имеются только действия сложения и вычитания. Эти действия называют действиями первой ступени.

    Выполняем действия слева направо по порядку (рис. 2).

    Рис. 2. Порядок действий

    Рассмотрим второе выражение

    В этом выражении имеются только действия умножения и деления – это действия второй ступени.

    Выполняем действия слева направо по порядку (рис. 3).

    Рис. 3. Порядок действий

    В каком порядке выполняются арифметические действия, если в выражении имеются не только действия сложения и вычитания, но и умножения и деления?

    Если в выражение без скобок входят не только действия сложения и вычитания, но и умножения и деления, или оба этих действия, то сначала выполняют по порядку (слева направо) умножение и деление, а затем сложение и вычитание.

    Рассуждаем так. В этом выражении имеются действия сложения и вычитания, умножения и деления. Действуем по правилу. Сначала выполняем по порядку (слева направо) умножение и деление, а затем сложение и вычитание. Расставим порядок действий.

    Вычислим значение выражения.

    Порядок выполнения арифметических действий в выражениях со скобками

    В каком порядке выполняются арифметические действия, если в выражении имеются скобки?

    Если в выражении имеются скобки, то сначала вычисляют значение выражений в скобках.

    Мы видим, что в этом выражении имеется действие в скобках, значит, это действие выполним первым, затем по порядку умножение и сложение. Расставим порядок действий.

    Вычислим значение выражения.

    Правило выполнения арифметических действий в выражениях без скобок и со скобками

    Как нужно рассуждать, чтобы правильно установить порядок арифметических действий в числовом выражении?

    Прежде чем приступить к вычислениям, надо рассмотреть выражение (выяснить, есть ли в нём скобки, какие действия в нём имеются) и только после этого выполнять действия в следующем порядке:

    1. действия, записанные в скобках;

    2. умножение и деление;

    3. сложение и вычитание.

    Схема поможет запомнить это несложное правило (рис. 4).

    Рис. 4. Порядок действий

    Выполнение тренировочных заданий на изученное правило

    Рассмотрим выражения, установим порядок действий и выполним вычисления.

    Будем действовать по правилу. В выражении 43 – (20 – 7) +15 имеются действия в скобках, а также действия сложения и вычитания. Установим порядок действий. Первым действием выполним действие в скобках, а затем по порядку слева направо вычитание и сложение.

    43 – (20 – 7) +15 =43 – 13 +15 = 30 + 15 = 45

    В выражении 32 + 9 * (19 – 16) имеются действия в скобках, а также действия умножения и сложения. По правилу первым выполним действие в скобках, затем умножение (число 9 умножаем на результат, полученный при вычитании) и сложение.

    32 + 9 * (19 – 16) =32 + 9 * 3 = 32 + 27 = 59

    В выражении 2*9-18:3 отсутствуют скобки, зато имеются действия умножения, деления и вычитания. Действуем по правилу. Сначала выполним слева направо умножение и деление, а затем от результата, полученного при умножении, вычтем результат, полученный при делении. То есть первое действие – умножение, второе – деление, третье – вычитание.

    Узнаем, правильно ли определен порядок действий в следующих выражениях.

    В этом выражении скобки отсутствуют, значит, сначала выполняем слева направо умножение или деление, затем сложение или вычитание. В данном выражении первое действие – деление, второе – умножение. Третье действие должно быть сложение, четвертое – вычитание. Вывод: порядок действий определен верно.

    Найдем значение данного выражения.

    Во втором выражении имеются скобки, значит, сначала выполняем действие в скобках, затем слева направо умножение или деление, сложение или вычитание. Проверяем: первое действие – в скобках, второе – деление, третье – сложение. Вывод: порядок действий определен неверно. Исправим ошибки, найдем значение выражения.

    В этом выражении также имеются скобки, значит, сначала выполняем действие в скобках, затем слева направо умножение или деление, сложение или вычитание. Проверяем: первое действие – в скобках, второе – умножение, третье – вычитание. Вывод: порядок действий определен неверно. Исправим ошибки, найдем значение выражения.

    Расставим порядок действий в выражении, используя изученное правило (рис. 5).

    Рис. 5. Порядок действий

    Мы не видим числовых значений, поэтому не сможем найти значение выражений, однако потренируемся применять изученное правило.

    Действуем по алгоритму.

    В первом выражении имеются скобки, значит, первое действие в скобках. Затем слева направо умножение и деление, потом слева направо вычитание и сложение.

    Во втором выражении также имеются скобки, значит, первое действие выполняем в скобках. После этого слева направо умножение и деление, после этого – вычитание.

    Проверим себя (рис. 6).

    Рис. 6. Порядок действий

    Сегодня на уроке мы познакомились с правилом порядка выполнения действий в выражениях без скобок и со скобками.

    Список литературы

    1. М. И. Моро, М.А. Бантова и др. Математика: Учебник. 3 класс: в 2-х частях, часть 1. – М.: «Просвещение», 2012.
    2. М.И. Моро, М.А. Бантова и др. Математика: Учебник. 3 класс: в 2-х частях, часть 2. – М.: «Просвещение», 2012.
    3. М.И. Моро. Уроки математики: Методические рекомендации для учителя. 3 класс. – М.: Просвещение, 2012.
    4. Нормативно-правовой документ. Контроль и оценка результатов обучения. – М.: «Просвещение», 2011.
    5. «Школа России»: Программы для начальной школы. – М.: «Просвещение», 2011.
    6. С.И. Волкова. Математика: Проверочные работы. 3 класс. – М.: Просвещение, 2012.
    7. В.Н. Рудницкая. Тесты. – М.: «Экзамен», 2012.

    Дополнительные рекомендованные ссылки на ресурсы сети Интернет

    Домашнее задание

    1. Определи порядок действий в данных выражениях. Найди значение выражений.

    2. Определи, в каком выражении такой порядок выполнения действий:

    1. умножение; 2. деление;. 3. сложение; 4. вычитание; 5. сложение. Найди значение данного выражения.

    3. Составь три выражения, в которых такой порядок выполнения действий:

    1. умножение; 2. сложение; 3. вычитание

    1. сложение; 2. вычитание; 3. сложение

    1. умножение; 2. деление; 3. сложение

    Найди значение этих выражений.

    Если вы нашли ошибку или неработающую ссылку, пожалуйста, сообщите нам – сделайте свой вклад в развитие проекта.

    Если несколько действий выполняются одно за другим, то результат, зависит от порядка действий.

    Если производить действия в порядке их записи.

    Если же сначала сложить 2 и 1 и вычесть полученную сумму из 4, то получим 1.

    Чтобы указать, в каком порядке нужно выполнять действия (в тех случаях, когда результат зависит от порядка действий), пользуются скобками. Действия, заключенные в скобки, выполняются раньше других. В нашем случае:

    Чтобы не загромождать чрезмерно записи, условились не писать скобок:

    1. в том случае, когда действия сложения и вычитания, следуя друг за другом, должны выполняться в том порядке, в каком они записаны;
    2. в том случае, когда внутри скобок производятся действия умножения или деления; например, вместо 2 + (4 · 5) = 22 пишут 2 + 4 · 5 = 22.

    При вычислении таких выражений, которые либо совсем не содержат скобок, либо содержат лишь такие скобки, внутри которых больше нет скобок, нужно производить действия в таком порядке:

    1. сначала выполняются действия, заключенные в скобки; при этом умножение и деление делаются в порядке из следования, но раньше, чем сложение и вычитание;
    2. затем выполняются остающиеся действия, причем опять умножение и деление делаются в порядке из следования, но раньше сложения и вычитания.

    Сначала выполняем умножения:
    2 · 5 = 10
    3 · 3 = 9
    затем вычитание:
    10 – 9 = 1

    Сначала выполняем действия в скобках:
    16 – 2 · 7 + 4 = 16 – 14 + 4 = 6
    2 + 5 = 7

    Теперь выполняем остающиеся действия:
    9 + 16 : 4 – 2 · 6 + 6 · 7 =
    = 9 + 4 – 12 + 42 =
    = 43

    Часто для указания порядка действий необходимо заключать в скобки такие выражения, которые сами уже содержат скобки. Тогда, кроме обычных (круглых), применяют скобки иной формы, например квадратные []. Если в скобки нужно заключить выражение, содержащее уже круглые и квадратные скобки, пользуются фигурными скобками <>. Вычисление подобных выражений производится в следующем порядке: сначала производятся вычисления внутри всех круглых скобок в вышеуказанной последовательности. Затем — вычисления внутри всех квадратных скобок по тем же правилам. Далее — вычисления внутри фигурных скобок и т.д.. Наконец, выполняются остающиеся действия.

    Выполняем действия в круглых скобках, имеем:
    8 – 6 = 2
    10 – 2 · 3 = 10 – 6 = 4

    действия в квадратных скобках дают:
    14 – 3 · 2 = 8

    выполняя остающиеся действия скобках находим:
    5 + 2 · 8 + 32 : 4 = 5 + 16 + 8 = 29

    Порядок действий:
    30 – 20 = 10
    35 – 10 = 25
    100 – 25 = 75
    75 · 2 = 150

    Порядок операций

    Когда у вас есть математическая задача, которая включает более одной операции, например, сложение и вычитание или вычитание и умножение ? Что вы делаете в первую очередь?

    Пример № 1 : 6? 3 х 2 =?

    • Вы делаете сначала вычитание (6? 3 = 3), а затем умножение (3 x 2 = 6 )?
    • Или вы начнете с умножения (3 x 2 = 6), а затем вычтите (6? 6 = 0 )?
    PEMDAS

    В подобных случаях мы следуем порядку операций . Порядок выполнения операций сокращен до PEMDAS :

    1. P arentheses
    2. E xponents
    3. M ultiplication и D ivision (слева направо)
    4. A ddition и S ubtraction (слева направо)

    (Один из способов запомнить это — вспомнить фразу P lease E xcuse M y D ear A Unt S союзник.)

    • В приведенном выше примере мы имеем дело с умножением и вычитанием. Ультипликация M идет на шаг перед убиранием S , поэтому сначала мы умножаем 3 x 2, а затем вычитаем сумму из 6, получая 0.

    Пример № 2 : 30 5 x 2 + 1 =?

    • Нет арентезов P .
    • Нет компонентов E .
    • Начнем с ultiplication M и ivision D , работая слева направо.
      ПРИМЕЧАНИЕ: Несмотря на то, что умножение предшествует делению в PEMDAS, они выполняются в одном шаге слева направо. Сложение и вычитание также выполняются на одном этапе.
    • 30 5 = 6 , в результате чего 6 x 2 + 1 =?
    • 6 x 2 = 12 , в результате чего 12 + 1 =?
    • Затем мы выполняем A ddition: 12 + 1 = 13

    Обратите внимание, что если бы мы выполняли умножение до деления, то получили бы неправильный ответ:

    • 5 x 2 = 10 , оставляя 30 10 + 1 =?
    • 30 10 = 3 , оставляя 3 + 1 =?
    • 3 + 1 = 4 (на 9! Меньше!)

    Последний пример для продвинутых студентов, использующий все шесть операций:

    Пример № 3 : 5 + (4? 2) 2 х 3 6? 1 =?

    • Начать с P аренцев: 4? 2 = 2 .(Хотя вычитание обычно выполняется на последнем шаге, потому что оно указано в скобках, мы делаем это в первую очередь.) Остается 5 + 2 2 x 3 6? 1 =?
    • Тогда E xponents: 2 2 = 4 . Теперь у нас 5 + 4 x 3 6? 1 =?
    • Затем M ultiplication и D ivision, начиная слева: 4 x 3 = 12 , оставляя нас с 5 + 12 6? 1 =?
    • Затем двигаемся вправо: 12 6 = 2 , что делает задачу 5 + 2? 1 =?
    • Затем A ddition и S ubtraction, начиная слева: 5 + 2 = 7 , оставляя 7? 1 =?
    • И, наконец, вправо: 7? 1 = 6

    (Для большей практики попробуйте нашу игру Operation Order!)


    Десятичные эквиваленты обыкновенных дробей Числа и формулы

    Десятичные эквиваленты обыкновенных дробей

    .com / ipa / 0/9/3/3/3/4 / A0933340.html

    Порядок операций — ChiliMath

    Фундаментальная концепция порядка операций — выполнять арифметические операции в «правильном» порядке или последовательности. Давайте посмотрим, как Роб и Пэтти пытались упростить данное числовое выражение, применяя порядок или правило операций.

    В чем ошибка Роба?

    • Он небрежно упростил числовые выражения, применив арифметические операции слева направо.

    Пэтти получила правильный ответ, потому что она правильно применила правила порядка операций.

    • Сначала она выполнила умножение и деление, а затем сложение и вычитание.

    Каков порядок действий?

    Порядок операций — это просто набор правил, устанавливающих приоритет последовательности операций , начиная от наиболее важных до наименее важных.

    Это правило о том, как правильно упрощать числовые выражения, также известно как правило PEMDAS (сокращение от P lease E xcuse M y D ear A Unt S ally).

    Шаг 1: Сделайте все возможное, чтобы упростить все, что находится внутри скобок или символа группировки.

    Шаг 2: По возможности упрощайте экспоненциальные числа в числовом выражении.

    Шаг 3: Умножение и деление слева направо в зависимости от того, что наступит раньше.

    Шаг 4: Сложите и вычтите в зависимости от того, что наступит раньше, слева направо


    Примеры применения порядка операций для упрощения числовых выражений

    Пример 1: Упростите приведенное ниже выражение, используя Порядок операций.

    • Рассматривая числовые выражения с несколькими операциями слева направо, мы видим, что сначала мы должны выполнить деление, что составляет 5 \ div 5 = 1.
    • На данный момент у меня есть три (3) возможных операции. В Порядке операций умножение имеет приоритет перед сложением и вычитанием. Следовательно, мы должны дальше размножаться. У нас 6 \ умножить на 2 = 12.
    • Что нам делать дальше, складывать или вычитать? В зависимости от порядка операций сложение и вычитание имеют одинаковое значение. Чтобы определить, какую операцию выполнить в первую очередь, мы добавляем или вычитаем слева направо в зависимости от того, что идет первым, что в этой ситуации должно сложить, 1 + 3 = 4.
    • Осталась одна операция — вычитание. На первый взгляд сложное числовое выражение сводится к окончательному ответу — 8.

    Пример 2: Упростите приведенное ниже выражение, используя Порядок операций.


    В следующих примерах будет задействовано скобок .Помните, что вам нужно сначала упростить все, что указано в скобках, прежде чем двигаться дальше.

    Пример 3: Упростите приведенное ниже выражение, используя Порядок операций.

    • Обратите внимание на выражения в скобках. Правило говорит нам сначала делить, а затем вычитать.
    • Мы можем избавиться от скобок, вычтя 7 на 2.
    • Умножение — это гораздо более «сильная» операция, чем вычитание, поэтому мы должны сначала умножить 5 и 4.
    • Закончите, вычтя 25 на 20.

    Пример 4: Упростите приведенное ниже выражение, используя Порядок операций.

    • Сначала упростите выражения в круглых скобках. Умножьте на первую скобку и разделите на вторую.
    • Сложите числа в первой скобке, затем вычтите числа внутри второй.
    • Здесь есть умножение и деление.Поскольку умножение предшествует делению, мы собираемся сначала умножить.
    • Между вычитанием и делением приоритет имеет деление, поэтому мы делим 5 на 5, чтобы получить 1.
    • Последняя оставшаяся операция — вычитание, поэтому мы и займемся этим.

    Последние примеры будут включать экспоненты, поэтому будьте осторожны на каждом шаге, потому что в них происходит очень много всего. Пока вы сосредотачиваетесь на соблюдении правил, регулирующих порядок действий, это не должно быть так сложно! Поехали…

    Пример 5: Упростите числовое выражение ниже, используя правила Порядка операций.

    • Упростите выражения в круглых скобках. Но, более конкретно, упростите числа с помощью показателей.
    • Для упрощения заключены две круглые скобки. Мы упростим второй, \ left ({30 — 27} \ right), потому что он намного проще. Здесь разница 30 и 27 составляет 3.
    • Теперь обратим внимание на другую скобку. Порядок действий говорит нам делить, прежде чем вычитать.
    • Наконец, мы можем избавиться от скобок, выполнив вычитание, потому что больше нечего делать.
    • Глядя на то, что мы оставили, упрощение экспоненциальных чисел имеет приоритет перед операциями умножения, сложения и вычитания.
    • Просматривая слева направо, очевидно, что мы должны умножать перед сложением и / или вычитанием.
    • В зависимости от порядка операций сложение и вычитание имеют одинаковое значение. Сначала мы должны вычесть, потому что операция вычитания предшествует сложению, если смотреть слева направо. 3} первый.
    • Заглянув в круглые скобки, мы должны сначала разделить, прежде чем умножать и вычитать.
    • Сохраняя внимание в круглых скобках, порядок операций говорит нам умножать, прежде чем мы будем вычитать.
    • Последняя операция внутри скобок — вычитание. Давай сделаем это!
    • Здесь сделаем паузу. На данный момент ясно, что мы можем выполнить три (3) упрощения одновременно.2}.
    • Кажущаяся сложной проблема теперь сведена к чему-то, что очень легко упростить. Если смотреть слева направо, деление имеет приоритет перед вычитанием и сложением.
    • Потому что вычитание и умножение находятся на одном уровне в иерархии операций. Способ, которым мы разрываем связь, как вы уже должны были знать, — это выполнять ту, которая идет первой, если смотреть слева направо. В этой ситуации мы будем вычитать, а затем прибавлять.Это оно!

    Практика с рабочими листами


    Возможно, вас заинтересует:

    Порядок действий Проблемы с ответами

    Правило PEMDAS

    Что такое порядок операций?

    Что такое порядок действий?

    В математике порядок операций — это правила, устанавливающие последовательность, в которой должны выполняться несколько операций в выражении.

    Способ запоминания порядка операций — PEMDAS, где каждая буква обозначает математическую операцию.

    п. Круглые скобки
    E Показатель
    M Умножение
    D Дивизия
    А Дополнение
    S Вычитание

    Правила PEMDAS, устанавливающие порядок, в котором должны выполняться операции в выражении, следующие:

    1. Круглые скобки — они имеют приоритет над всеми другими операторами. Первый шаг — выполнить все операции в скобках. Проработайте все группировки изнутри наружу. (Все, что указано в скобках, является группировкой)

    2. Экспоненты — Найдите все экспоненциальные выражения.

    3. Умножение и деление — Затем, двигаясь слева направо, умножайте и / или делите в зависимости от того, что наступит раньше. 4. Сложение и вычитание — Наконец, двигаясь слева направо, складывайте и / или вычитайте в зависимости от того, что наступит раньше.

    Почему следует соблюдать порядок действий?

    Следуйте правилам порядка операций для решения выражений, чтобы все пришли к одному и тому же ответу.

    Вот пример того, как мы можем получить разные ответы, если НЕ соблюдаем правильный порядок операций.

    Выражение решено слева направо Выражение решено с использованием порядка операций (PEMDAS)

    6 х 3 + 4 х (9 ÷ 3)

    6 х 3 + 4 х (9 ÷ 3)

    18 + 4 x (9 ÷ 3)

    22 х (9 ÷ 3)

    198 ÷ 3

    = 66 ✘

    6 х 3 + 4 х (9 ÷ 3)

    6 х 3 + 4 х (9 ÷ 3) пол.

    6 х 3 + 4 х 3 → М

    18 + 4 x 3 → М

    18 + 12 → А

    = 30 ✔

    Интересные факты

    • Популярная мнемоника, используемая для запоминания порядка действий — PEMDAS — это «Прошу прощения, моя дорогая тетя Салли».

    Давайте споем!

    Все дело в операциях,

    Решайте по порядку, иначе будет напряженность.

    Начните с открытия скобок.

    Прыгайте с экспонентами.

    Куб или Квадрат — это все очень честно!

    Далее, Умножение или Разделение — переход слева направо.

    Сложение и вычитание идут последними, но они просты.

    наконец, это так просто, как A B C D!

    Давайте сделаем это!

    Вместо того, чтобы раздавать ребенку рабочие листы, составляйте словесные задачи из реальных жизненных ситуаций.

    Это поможет им писать и решать выражения, а также использовать порядок операций для упрощения выражений в предалгебре и алгебре.

    Например, возьмите ребенка за покупками. Попросите их выбрать 2 дюжины яиц, 3 пакета булочек для хот-догов, 2 пакета конфет и 2 коробки хлопьев.Затем попросите их положить обратно одну коробку хлопьев. Теперь спросите у ребенка количество яиц в дюжине, количество булочек в пачке, количество конфет в пачке и подсчитайте общее количество купленных предметов. Попросите их составить выражение и использовать порядок действий, чтобы найти ответ.

    Сопутствующий математический словарь

    Правило PEMDAS: понимание порядка операций

    Каждый, кто посещал математические курсы в США, раньше слышал аббревиатуру «PEMDAS».Но что именно это означает? Здесь мы подробно объясним значение PEMDAS и то, как он используется , прежде чем дать вам несколько примеров задач PEMDAS, чтобы вы могли практиковать то, что вы узнали.

    PEMDAS Значение: что это означает?

    PEMDAS — это аббревиатура, призванная помочь вам запомнить порядок операций, используемых для решения математических задач. Это обычно произносится как «пем-дасс», «пем-дозз» или «пем-досс».

    Вот что означает каждая буква в PEMDAS:

    • P аренцев
    • E xponents
    • M ultiplication и D ivision
    • A ddition и S ubtraction

    Порядок букв показывает порядок, в котором вы должны решать различные части математической задачи , причем выражения в скобках идут первыми, а сложение и вычитание — последними.

    Многие ученики используют этот мнемонический прием, чтобы помочь им запомнить каждую букву: Пожалуйста, извините, моя дорогая тетя Салли .

    В Великобритании и других странах студентов обычно изучают PEMDAS как BODMAS . Значение BODMAS такое же, как значение PEMDAS — просто используется пара разных слов. В этом аббревиатуре B обозначает «скобки» (то, что мы в США называем круглыми скобками), а O обозначает «порядки» (или показатели).

    Итак, как именно вы используете правило PEMDAS? Давайте взглянем.

    Как вы используете PEMDAS?

    PEMDAS — это аббревиатура, используемая для напоминания людям о порядке операций.

    Это означает, что вы не просто решаете математические задачи слева направо; скорее, вы решаете их в заранее определенном порядке, который дается вам через аббревиатуру PEMDAS . Другими словами, вы начнете с упрощения любых выражений в круглых скобках, прежде чем упрощать любые показатели и переходить к умножению и т. Д.

    Но это еще не все.Вот что означает PEMDAS для решения математических задач:

    • Круглые скобки: Все, что указано в скобках, необходимо сначала упростить
    • Показатели: Все, что имеет показатель степени (или квадратный корень), должно быть упрощено после все в скобках было упрощено
    • Умножение и деление: После того, как разобрались со скобками и показателями степени, решите любое умножение и деление слева направо
    • Сложение и вычитание: После того, как разобрались со скобками, экспонентами, умножением и делением, решите любое сложение и вычитание слева направо

    Если какой-либо из этих элементов отсутствует (например,g. , у вас есть математическая задача без показателей), вы можете просто пропустить этот шаг и перейти к следующему.

    Теперь давайте рассмотрим пример задачи, чтобы помочь вам лучше понять правило PEMDAS:

    4 (5 — 3) ² — 10 ÷ 5 + 8

    У вас может возникнуть соблазн решить эту математическую задачу слева направо, но это приведет к неправильному ответу! Итак, вместо этого давайте использовать PEMDAS, чтобы помочь нам приблизиться к правильному подходу к .

    Мы знаем, что сначала нужно разобраться со скобками.В этой задаче заключены одни скобки: (5 — 3). Упрощение дает 2 , поэтому теперь наше уравнение выглядит так:

    4 (2) ² — 10 ÷ 5 + 8

    Следующая часть PEMDAS — экспоненты (и квадратные корни). В этой задаче есть один показатель степени, который возводит в квадрат число 2 (то есть то, что мы нашли, упростив выражение в скобках).

    Это дает нам 2 × 2 = 4. Итак, теперь наше уравнение выглядит так:

    4 (4) — 10 ÷ 5 + 8 ИЛИ 4 × 4 — 10 ÷ 5 + 8

    Далее идет умножение и деление слева направо . Наша задача содержит как умножение, так и деление, которые мы будем решать слева направо (сначала 4 × 4, а затем 10 ÷ 5). Это упрощает наше уравнение следующим образом:

    16-2 + 8

    Наконец, все, что нам нужно сделать, это решить оставшееся сложение и вычитание слева направо :

    16-2 + 8
    14 + 8
    = 22

    Окончательный ответ: 22. Не верите? Вставьте все уравнение в свой калькулятор (написанное в точности так, как указано выше), и вы получите тот же результат!

    Дэвид Геринг / Flickr

    Примеры математических задач с использованием PEMDAS + ответы

    Посмотрите, сможете ли вы правильно решить следующие четыре проблемы, используя правило PEMDAS.Мы рассмотрим ответы позже.

    Пример проблем PEMDAS
    1. 11-8 + 5 × 6
    2. 8 ÷ 2 (2 + 2)
    3. 7 × 4 — 10 (5 — 3) ÷ 2²
    4. √25 (4 + 2) ² — 18 ÷ 3 (3 — 1) + 2³
    Ответы
    1. 33
    2. 16
    3. 23
    4. 176
    Ответ объяснения

    Здесь мы рассмотрим каждую проблему, указанную выше, и то, как вы можете использовать PEMDAS, чтобы получить правильный ответ.

    # 1 Объяснение ответа

    11-8 + 5 × 6

    Эта математическая задача представляет собой довольно простой пример PEMDAS, который использует сложение, вычитание и умножение только , поэтому здесь не нужно беспокоиться о скобках или показателях степени.

    Мы знаем, что умножение предшествует сложению и вычитанию , поэтому вам нужно начать с умножения 5 на 6, чтобы получить 30:

    .

    11–8 + 30

    Теперь мы можем просто работать слева направо над сложением и вычитанием:

    11-8 + 30
    3 + 30
    = 33

    Это приводит нас к , правильный ответ — 33 .

    # 2 Объяснение ответа

    8 ÷ 2 (2 + 2)

    Если эта математическая задача кажется вам знакомой, вероятно, это связано с тем, что стал вирусным в августе 2019 года из-за своей неоднозначной настройки . Многие люди спорили о том, был ли правильный ответ 1 или 16, но, как все мы знаем, в математике есть (почти всегда!) Только один истинно правильный ответ .

    Так что это: 1 или 16?

    Давайте посмотрим, как PEMDAS может дать нам правильный ответ.В этой задаче есть скобки, деление и умножение. Итак, мы начнем с упрощения выражения в скобках, согласно PEMDAS:

    .

    8 ÷ 2 (4)

    Хотя большинство людей в сети до этого момента соглашались, многие не соглашались с тем, что делать дальше: умножить ли 2 на 4 или разделить 8 на 2?

    PEMDAS может ответить на этот вопрос: когда дело доходит до умножения и деления, вы всегда работаете слева направо. Это означает, что вы действительно должны разделить 8 на 2, прежде чем умножить на 4.

    Было бы полезно взглянуть на проблему таким образом, поскольку люди склонны запутаться в круглых скобках (помните, что все, что находится рядом с круглыми скобками, это , умноженное на на значение, указанное в скобках):

    8 ÷ 2 × 4

    Теперь решим уравнение слева направо:

    8 ÷ 2 × 4
    4 × 4
    = 16

    Правильный ответ — 16. Любой, кто утверждает, что это 1, определенно неправ — и явно неправильно использует PEMDAS!

    Если бы только эти примеры проблем PEMDAS были такими простыми…

    # 3 Ответ Объяснение

    7 × 4 — 10 (5 — 3) ÷ 2²

    Теперь все становится немного сложнее.

    В этой математической задаче есть скобки, показатель степени, умножение, деление, вычитание и . Но не расстраивайтесь — давайте поработаем над уравнением, шаг за шагом.

    Во-первых, согласно правилу PEMDAS, мы должны упростить то, что в скобках :

    7 × 4 — 10 (2) ÷ 2²

    Легко и просто, правда? Затем давайте упростим показатель степени :

    7 × 4 — 10 (2) ÷ 4

    Теперь осталось только умножение, деление и вычитание.Помните, что с умножением и делением мы просто работаем слева направо:

    7 × 4-10 (2) ÷ 4
    28-10 (2) ÷ 4
    28-20 ÷ 4
    28-5

    После того, как вы умножили и разделили, вам просто нужно , выполнить вычитание , чтобы решить его:

    28–5
    = 23

    Это дает нам правильный ответ 23 .

    # 4 Объяснение ответа

    √25 (4 + 2) ² — 18 ÷ 3 (3 — 1) + 2³

    Эта проблема может показаться пугающей, но я обещаю, что это не так! Если вы подходите к ней по одному шагу за раз, используя правило PEMDAS , вы сможете решить ее в кратчайшие сроки.

    Сразу видно, что эта задача содержит все компонента PEMDAS : круглые скобки (два набора), показатели степени (два и квадратный корень), умножение, деление, сложение и вычитание. Но на самом деле это не отличается от любой другой математической задачи, которую мы решали.

    Во-первых, мы должны упростить то, что заключено в два набора круглых скобок:

    √25 (6) ² — 18 ÷ 3 (2) + 2³

    Затем мы должны упростить все экспоненты — , включая квадратные корни :

    5 (36) — 18 ÷ 3 (2) + 8

    Теперь мы должны произвести умножение и деление слева направо:

    5 (36) — 18 ÷ 3 (2) + 8
    180-18 ÷ 3 (2) + 8
    180-6 (2) + 8
    180-12 + 8

    Наконец, решаем оставшееся сложение и вычитание слева направо:

    180 — 12 + 8
    168 + 8
    = 176

    Это приводит нас к и правильному ответу 176 .

    Что дальше?

    Еще одна математическая аббревиатура, которую вам следует знать — SOHCAHTOA. В нашем экспертном руководстве рассказывается, что означает аббревиатура SOHCAHTOAH и как вы можете использовать его для решения задач, связанных с треугольниками.

    Готовитесь к разделу SAT или ACT Math? Тогда вы обязательно захотите ознакомиться с нашим полным руководством по SAT Math / ACT Math, которое дает вам множество советов и стратегий для этого сложного раздела.

    Заинтересованы в действительно больших цифрах? Узнайте, что такое гугол и гуголплекс, а также почему невозможно выписать одно из этих чисел.

    Что такое PEMDAS? — Определение, правила и примеры — Видео и стенограмма урока

    Почему PEMDAS важен?

    Без PEMDAS нет указаний для получения только одного правильного ответа. В качестве очень простого примера, чтобы вычислить 2 * 4 + 7, я мог бы сначала умножить, а затем сложить, чтобы получить 15. У меня также есть возможность сначала сложить, а затем умножить и получить 22. Какой ответ правильный? Используя PEMDAS, единственный правильный ответ — 15, потому что порядок букв в PEMDAS говорит мне, что умножение M должно выполняться перед сложением A.

    Вот объяснение правил, приведенных в PEMDAS:

    1. P, поскольку первая буква означает, что вы сначала выполняете любые вычисления с группировкой символов.
    2. Затем найдите показатели степени, E. Игнорируйте любые другие операции и возьмите любые числа с показателями степени в соответствующие степени.
    3. Несмотря на то, что M для умножения в PEMDAS стоит перед D для деления, эти две операции фактически имеют одинаковый приоритет. Выполняйте только эти две операции в порядке их следования слева направо.2) + 10
      • 36 — 2 (20 + 12/4 * 3-4) + 10
        • 36 — 2 (20 + 3 * 3-4) + 10
    4. Игнорируя сложение и вычитание, я завершаю следующую операцию умножения. 2} + 12/4.3 |, шаги будут следующими:

      Краткое содержание урока

      PEMDAS — это аббревиатура слов скобка, показатель степени, умножение, деление, сложение, вычитание. Для любого выражения сначала следует упростить все показатели, затем умножить и разделить слева направо и, наконец, сложить и вычесть слева направо. Слово «круглые скобки» стоит первым в этом аббревиатуре, чтобы указать, что любое выражение в символе группировки, такое как круглые скобки, должно быть сначала упрощено.Этот приказ также можно запомнить, используя фразу «Прошу прощения, моя дорогая тетя Салли».

      Результаты обучения

      Изучив этот урок на PEMDAS, откройте для себя свою способность:

    • Осознайте важность PEMDAS и произнесите фразу, которая поможет вам запомнить порядок действий
    • Использование PEMDAS в математических выражениях
    • Понять, как PEMDAS применяется к выражениям дробей и абсолютных значений

    Порядок операций

    Поделитесь этой страницей!

    После изучения 4 основных операций сложения, вычитания, умножения и деления пора научиться использовать их в различных комбинациях. Чтобы сделать это правильно, нам нужно понять некоторые правила того, как операции соотносятся друг с другом.

    Мы делаем это, понимая порядок важности каждой операции относительно других.

    Правило: сложение и вычитание одинаково важны.

    Это означает, что мы выполняем сложение и / или вычитание в том порядке, в котором они появляются (слева направо).

    Пример 1

    10 + 4-8
    = 14-8 (тренировка 10 + 4 вначале)

    = 6

    Пример 2

    10-7 + 8
    = 3 + 8 (сначала тренировка 10-7)

    = 11

    Если вы выполните расчет в другом порядке, в большинстве случаев вы получите другой результат.Так что навести порядок в правильной последовательности жизненно важно.

    Пусть ваш ребенок или ученик сначала поработает с двумя операциями, прежде чем переходить к более сложным выражениям, включающим три, четыре или более операций.

    Правило: умножение и деление имеют одинаковое значение.

    Это означает, что мы выполняем умножение и / или деление в том порядке, в котором они появляются (слева направо).

    Правило: умножение и деление важнее сложения и вычитания.

    Это означает, что нам нужно сначала выполнить умножение и деление, прежде чем мы сможем выполнять сложение и вычитание.

    Правило: круглые скобки или квадратные скобки () более важны, чем каждая из 4 операций.

    Это означает, что мы должны выполнить все, что находится в круглых скобках (), прежде чем следовать другим правилам.

    Примеры

    Это приводит нас к правилу BODMAS :

    Скобки, порядки (степени и корни), деление и умножение, сложение и вычитание

    Или правило PEMDAS :

    Круглые скобки, экспоненты, умножение и деление и сложение и вычитание

    Для облегчения запоминания: Прошу прощения, моя дорогая тетя Салли

    Практические вопросы

    ОТВЕТЫ

    Термины для уравнений сложения, вычитания, умножения и деления — математика для 3-го класса

    Изучите термины для уравнений сложения, вычитания, умножения и деления

    Итак, вы научились решать уравнения сложения, вычитания, умножения и деления. 👏

    Давайте рассмотрим терминов для каждого из них.

    Совет: Термины — это имен различных частей уравнения.

    Условия добавления

    Слагаемые — это числа, которые складываются вместе.

    Сумма — это ответ, который вы получите, сложив числа.

    Мы пишем плюс ( +) между двумя слагаемыми и знак равенства перед суммой.

    Совет: Знак равенства (=) означает, что элементы слева и справа от него равны.

    Термины для вычитания

    Minuend — это число, из которого вычитается. Это большее число.

    Subtrahend — это число, которое убирается из убываемого. Это меньшее число.

    Вычитаемое всегда предшествует вычитаемому.

    Наконечник для запоминания:

    Разница — это ответ, который мы получаем в уравнении вычитания.

    Мы используем знак минус (-) между минусом и вычитаемым.

    Запишем знак равенства перед разностью.

    Условия умножения

    Умножаемое — это число, которое нужно умножить.

    Умножитель — это число, указывающее, сколько раз следует умножить множимое.

    Множаемое и множитель также называются коэффициентами .

    Множитель часто записывается первым, но положение этих чисел не имеет особого значения. Это называется коммутативным свойством умножения.

    Ответ в уравнении умножения называется произведением .

    Знак умножения ( ×) записывается между двумя множителями. Его также называют знаком раз.

    Условия для подкласса

    Дивиденд — это делимое число.

    Делитель — это число, которое указывает, сколько раз следует разделить дивиденд.Он отвечает на вопрос «На сколько равных групп делится число?».

    Ответ, который мы получаем в уравнении деления, называется частным .

    Знак деления (÷) помещается между делимым и делителем. Это короткая горизонтальная линия с точками над и под ней.

    Совет: Вы также можете увидеть /, используемые как знак деления. То же, что и ÷.

    Смотри и учись

    Отличная работа по изучению этих терминов.👏

    А теперь попробуйте практиковаться, чтобы убедиться, что вы помните, что они означают.

    .

    Какое действие производится первым сложение или умножение?

    Содержание

    • — Что в примере делается первым?
    • — Как решать примеры со скобками и умножением?
    • — Почему сначала идет умножение?
    • — Что важнее скобки или умножение?
    • — Что сначала возведение в степень или умножение?
    • — Что значит выражение в математике?
    • — Что первое умножение или деление со скобками?
    • — Что делать сначала сложение или вычитание?
    • — Как считать пример с квадратными скобками?
    • — Какой правильный ответ 6 2 1 2?
    • — Что такое сложение и вычитание?
    • — Как умножать в столбик?

    Есть однозначное правило, которое определяет порядок выполнения действий в выражениях без скобок: действия выполняются по порядку слева направо сначала выполняется умножение и деление, а затем — сложение и вычитание.

    Что в примере делается первым?

    Сначала умножение и деление, затем сложение и вычитание В школе дается следующее правило, определяющее порядок выполнения действий в выражениях без скобок: действия выполняются по порядку слева направо, причем сначала выполняется умножение и деление, а затем – сложение и вычитание.

    Как решать примеры со скобками и умножением?

    Запомните правило:

    1. Если в примере нет скобок, сначала выполняем действия умножения и деления по порядку, слева направо. …
    2. Если в примере есть скобки, то сначала мы выполняем действия в скобках, затем умножение и деление, и затем — сложение и вычитание начиная слева направо.

    Почему сначала идет умножение?

    Так всё же почему умножение выполняется первое, а только потом сложение? Ответ довольно прост. При умножении двух разных единиц измерения получается новая единица измерения, при сложении единицы измерения не меняются. При умножении мы получаем эту самую новую единицу измерения.

    Что важнее скобки или умножение?

    сначала выполняются действия, заключенные в скобки; при этом умножение и деление делаются в порядке из следования, но раньше, чем сложение и вычитание; затем выполняются остающиеся действия, причем опять умножение и деление делаются в порядке из следования, но раньше сложения и вычитания.

    Что сначала возведение в степень или умножение?

    Для основных арифметических действий установлен следующий порядок: сначала выполняется возведение числа в степень, затем выполняется умножение и деление и в самую последнюю очередь выполняется сложение и вычитание.

    Что значит выражение в математике?

    Числовое выражение – это запись, состоящая из чисел и знаков действий между ними. Значение выражения – это результат выполненных действий. Сравнить числовые выражения – найти значение каждого из выражений и их сравнить. Порядок выполнения действий – это последовательность проводимых вычислений в данном выражении.

    Что первое умножение или деление со скобками?

    Сначала выполнить действия в скобках, при этом также по порядку слева направо выполняется умножение и деление, затем — сложение и вычитание. Выражения в скобках рассматриваются как составные части исходного выражения. В них сохраняется уже известный нам порядок выполнения действий.

    Что делать сначала сложение или вычитание?

    Действия в числовых выражениях выполняются в следующем порядке: Действия записанные в скобках; Умножение иделение по порядку: слева направо; Сложение и вычитание по порядку: слева направо.

    Как считать пример с квадратными скобками?

    По правилу всегда выполняется первым действие в скобках, после чего умножение и деление, а позже сложение и вычитание. Рассмотрим на примере заданное выражение. Если дан пример вида 5+3-2, тогда очевидно, что действия выполняются последовательно.

    Какой правильный ответ 6 2 1 2?

    Правильный ответ «9».

    Что такое сложение и вычитание?

    Сложение – это объединение объектов в одно целое. Результатом сложения чисел является число, называемое суммой чисел (слагаемых). Вычитание – это такое действие, в котором отнимают меньшее число от большего. Большее число называется уменьшаемым, меньшее – вычитаемым, результат вычитания – разностью.

    Как умножать в столбик?

    Умножение в столбик

    1. Запишем числа столбиком (одно под другим). В верхней строчке — большее число, в нижней — меньшее. …
    2. Сначала умножаем целиком верхнее число на последнюю цифру нижнего числа. …
    3. Умножаем «2» на «6». …
    4. Умножаем «4» на «6». …
    5. Переходим к умножению числа «427» на «3».

    Интересные материалы:

    Какие есть энциклопедии?
    Какие есть евреи в Израиле?
    Какие есть фильмы страшилки?
    Какие есть форм факторы материнских плат?
    Какие есть форматы для электронных книг?
    Какие есть форматы для фото?
    Какие есть формы обучения в колледже?
    Какие есть формы познания?
    Какие есть формы проведения мероприятий?
    Какие есть фразеологические словари?

    Как обманывают в школе

    Учителя врут школьникам. И это факт. Когда бывшие школьники начинают изучать высшую математику в университете, они с ужасом узнают, что им в школе говорили, мягко говоря, не правду. И хорошо, если квалификация школьного учителя была достаточно высокой и у него хватало умения и терпения отвечать на сложные вопросы, не навязывая не правильное понимание предмета, а оставляя школьнику возможность самостоятельно размышлять. Сложившиеся в школе стереотипы долго потом еще преследуют студента и ему приходится ломать себя. Остановимся на одной такой нелепости, активно рекламируемой в школе:

    ДЕЛИТЬ НА НОЛЬ НЕЛЬЗЯ


    Что значит 5 – 3? Надо взять пять предметов, отнять (убрать) три из них. Но математики смотрят на эту задачу иначе. Нет никакого вычитания, есть только сложение. Поэтому 5 – 3 означает такое число, которое при сложении с числом 3 даст число 5. То есть 5 – 3 — это сокращенная запись уравнения: x + 3 = 5. В этом уравнении нет никакого вычитания. Есть только задача — найти подходящее число.

    Странная, вообще говоря, позиция: «нет никакого вычитания». Может и нет, но это еще больше запутает школьника. Но оказывается еще математики не знают отрицательных чисел и сложение положительного и отрицательного: 5+(-3) для математиков понятие не постижимое. Зато знают уравнения, которые якобы можно решать без отрицательных чисел, а просто так «подбором подходящего числа». Как потом учитель будет объяснять как решаются уравнения трудно даже представить. Дальше, там же, анализируется деление с тех же позиций тех же якобы математиков.

    Точно так же обстоит дело с умножением и делением. Запись 8 : 4 можно понимать как результат разделения восьми предметов по четырем равным кучкам. Но в действительности это просто сокращенная форма записи уравнения 4 · x = 8. Вот тут-то и становится ясно, почему нельзя (а точнее невозможно) делить на ноль. Запись 5 : 0 — это сокращение от 0 · x = 5. То есть это задание найти такое число, которое при умножении на 0 даст 5. Но мы знаем, что при умножении на 0 всегда получается 0. Это неотъемлемое свойство нуля, строго говоря, часть его определения. Такого числа, которое при умножении на 0 даст что-то кроме нуля, просто не существует. То есть наша задача не имеет решения. А значит, записи 5 : 0 не соответствует никакого конкретного числа, и она просто ничего не обозначает и потому не имеет смысла. Бессмысленность этой записи кратко выражают, говоря, что на ноль делить нельзя.

    Особенно тут радует фраза: «Это неотъемлемое свойство нуля, строго говоря, часть его определения». А никто и не собирается у нуля отнимать его свойства. Ну и дальше, уже для особо одаренных особый случай, что явно не порадует и не добавит понимания школьнику.

    А можно ли ноль делить на ноль? В самом деле, ведь уравнение 0 · x = 0 благополучно решается. Например, можно взять x = 0, и тогда получаем 0 · 0 = 0. Выходит, 0 : 0=0? Но не будем спешить. Попробуем взять x = 1. Получим 0 · 1 = 0. Правильно? Значит, 0 : 0 = 1? Но ведь так можно взять любое число и получить 0 : 0 = 5, 0 : 0 = 317 и т. д. Но если подходит любое число, то у нас нет никаких оснований остановить свой выбор на каком-то одном из них. То есть мы не можем сказать, какому числу соответствует запись 0 : 0. А раз так, то мы вынуждены признать, что эта запись тоже не имеет смысла.

    Такое объясненение, хоть и содержит ляпы, но все же лучше, чем просто утверждение, что делить нельзя и все. Тем не менее, это не наилучший вариант объяснения для школьников, которым надо понять еще и смысл уравнения:

    $$x+3=5,$$

    в котором фигурирует непонятный х. Студенческая лаборатория предлагает свой вариант объяснения проблемы деления на ноль. И этот вариант будет понятен не только учителю, но и ученику.

    Умножение можно заменить многократным сложением. Например, вместо умножения четверки на двойку можно четыре раза сложить двойку:

    $$4\cdot2=2+2+2+2=8$$
    А деление можно заменить вычитанием. Вместо деления будем из восьмерки четыре раза вычитать двойку пока не получится ноль. Сколько раз вычитали двойку, тому и равен результат деления. В данном случае это было сделано четыре раза: $$\frac{8}{2}\Rightarrow 8-2-2-2-2=0$$ А теперь рассмотрим случай деления на ноль. Следуя предыдущему правилу замены деления вычитанием, необходимо из делимого вычитать делитель (ноль) до тех пор пока не получится ноль. А это не случится никогда: $$\frac{8}{0}\Rightarrow 8-0-0-0-0….=?$$ Вот почему число на ноль разделить не получится. Очень коротко, понятно, просто. Если же ноль надо разделить на ноль, то ноль из нуля можно вычитать любое число раз, а значит результат будет любым. И не надо при этом говорить, что в этом случае деление не возможно. Лучше честно сказать, что этим случаем занимается высшая математика. Но объяснение можно все-таки привести (смотри ниже) и оно будет понятно школьнику. Кстати, именно это объснение позволит построить мостик к пониманию бесконечности, о которой говорят уже студентам в курсе высшей математики, когда приходится делить на ноль, а точнее на величину бесконечно близкую к нулю. Хоть эта величина и близка к нулю, но все таки она не равна нулю, а только стремится к нулю: \(\varepsilon \rightarrow 0\). И если из некоторого числа (например, 8-ми) вычитать эту бесконечно малую величину, то ее придется вычесть бесконечное (или очень большое) число раз, чтобы делимое, из которого вычитают, приблизилось к нулю:
    $$\frac{8}{0}\Rightarrow 8-\varepsilon -\varepsilon -\varepsilon -\varepsilon ….-\varepsilon \rightarrow 0$$
    Поэтому и результат деления восьмерки на ноль (или почти ноль) будет равен бесконечности (очень большому числу — для школьников).

    В этом объяснении никаких уравнений, никаких иксов, а понятные и доступные любому школьнику арифметические операции: сложение и вычитание. И, самое главное, в этом объяснении — правда и подготовка к пониманию высшей математики. Принимаем критику, комментарии и ваши варианты.

    © Цитирование и копирование только со ссылкой на Studlab.com.

    Studlab. com — Студенческая лаборатория

    Умножение деление пятое действие. Порядок выполнения действий в выражениях без скобок и со скобками

    Видеоурок «Порядок выполнения действий» подробно поясняет важную тему математики — последовательность выполнения арифметических операций при решении выражения. В ходе видеоурока рассматривается, какой приоритет имеют различные математические операции, как это применяется в вычислении выражений, приводятся примеры для усвоения материала, обобщаются полученные знания в решении заданий, где имеются все рассмотренные операции. С помощью видеоурока учитель имеет возможность быстрее достичь целей урока, повысить его эффективность. Видео может применяться в качестве наглядного материала, сопровождающего объяснение учителя, а также в качестве самостоятельной части урока.

    В наглядном материале используются приемы, которые помогают лучше достичь понимания темы, а также запомнить важные правила. С помощью цвета и разного написания выделяются особенности и свойства операций, отмечаются особенности решения примеров. Анимационные эффекты помогают подавать последовательно учебный материал, а также обратить внимание учеников на важные моменты. Видео озвучено, поэтому дополняется комментариями учителя, помогающими ученику понять и запомнить тему.

    Видеоурок начинается с представления темы. Затем отмечается, что умножение, вычитание являются операциями первой ступени, операции умножения и деления названы операциями второй ступени. Данным определением нужно будет оперировать дальше, выведено на экран и выделено цветным крупным шрифтом. Затем представляются правила, составляющие порядок выполнения операций. Выводится первое правило порядка, которое указывает, что при отсутствии скобок в выражении, наличию действий одной ступени, данные действия необходимо производить по порядку. Во втором правиле порядка утверждается, что при наличии действий обеих ступеней и отсутствии скобок, производятся первыми операции второй ступени, потом производятся операции первой ступени. Третье правило устанавливает порядок выполнения операций, для выражений, включающих скобки. Отмечается, что в этом случае сначала производятся операции в скобках. Формулировки правил выделены цветным шрифтом и рекомендованы к запоминанию.

    Далее предлагается усвоить порядок выполнения операций, рассматривая примеры. Описывается решение выражения с содержанием только операций сложения, вычитания. Отмечаются основные особенности, которые влияют на порядок вычислений — отсутствуют скобки, присутствуют операции первой ступени. Ниже расписано по действиям, как выполняются вычисления, сначала вычитание, затем два раза сложение, а затем вычитание.

    Во втором примере 780:39·212:156·13 требуется вычислить выражение, выполняя действия согласно порядку. Отмечается, что в данном выражении содержатся исключительно операции второй ступени, без скобок. В данном примере все действия производятся строго слева направо. Ниже поочередно расписываются действия, постепенно подходя к ответу. В результате вычисления получается число 520.

    В третьем примере рассматривается решение примера, в котором есть операции обеих ступеней. Отмечается, что в данном выражении отсутствуют скобки, но есть действия обеих ступеней. Согласно порядку выполнения операций, производятся операции второй ступени, после этого — операции первой ступени. Ниже — по действиям расписывается решение, в котором выполняются сначала три операции — умножение, деление, еще одно деление. Затем с найденными значениями произведения и частных производятся операции первой ступени. В ходе решения фигурными скобками объединены действия каждой ступени для наглядности.

    В следующем примере содержатся скобки. Поэтому демонстрируется, что первые вычисления производятся над выражениями в скобках. После них производятся операции второй ступени, следом — первой.

    Далее представлено замечание о том, в каких случаях можно не записывать скобки при решении выражений. Замечено, что это возможно только в случае, когда устранение скобок не изменить порядок выполнения операций. Примером служит выражение со скобками (53-12)+14, которое содержит только операции первой ступени. Переписав 53-12+14 с устранением скобок, можно отметить, что порядок поиска значения не изменится — сначала выполняется вычитание 53-12=41, а затем сложение 41+14=55. Ниже отмечается, что менять порядок операций при нахождении решения выражения можно, используя свойства операций.

    В конце видеоурока изученный материал обобщается в выводе, что каждое выражение, требующее решения, задает определенную программу для вычисления, состоящую из команд. Пример такой программы представляется при описании решения сложного примера, представляющего собой частное (814+36·27) и (101-2052:38). Заданная программа содержит пункты: 1) найти произведение 36 с 27, 2) добавить к 814 найденную сумму, 3) поделить на 38 число 2052, 4) отнять из числа 101 результат деления 3 пункта, 5) поделить результат выполнения пункта 2 на результат пункта 4.

    В конце видеоурока представлен перечень вопросов, на которые предлагается ответить ученикам. В их числе умение отличить действия первой и второй ступеней, вопросы о порядке выполнения действий в выражениях с действиями одной ступени и разных ступеней, о порядке выполнения действий при наличии скобок в выражении.

    Видеоурок «Порядок выполнения действий» рекомендуется применять на традиционном школьном уроке для повышения эффективности урока. Также наглядный материал будет полезен для проведения дистанционного обучения. Если ученику необходимо дополнительное занятие для освоения темы или он изучает ее самостоятельно, видео может быть рекомендовано для самостоятельного изучения.

    Мы рассмотрим в этой статье три варианта примеров:

    1. Примеры со скобками (действия сложения и вычитания)

    2. Примеры со скобками (сложение, вычитание, умножение, деление)

    3. Примеры, в которых много действий

    1 Примеры со скобками (действия сложения и вычитания)

    Рассмотрим три примера. В каждом из них порядок действий обозначен цифрами красного цвета:

    Мы видим, что порядок действий в каждом примере будет разный, хотя числа и знаки одинаковые. Это происходит потому, что во втором и третьем примере есть скобки.

    *Это правило для примеров без умножения и деления. Правила для примеров со скобками, включающих действия умножения и деления мы рассмотрим во второй части этой статьи.

    Чтобы не запутаться в примере со скобками, можно превратить его в обычный пример, без скобок. Для этого результат, полученный в скобках, записываем над скобками, далее переписываем весь пример, записывая вместо скобок этот результат, и далее выполняем все действия по порядку, слева направо:

    В несложных примерах можно все эти операции производить в уме. Главное — сначала выполнить действие в скобках и запомнить результат, а затем считать по порядку, слева направо.

    А теперь — тренажеры!

    1) Примеры со скобками в пределах до 20. Онлайн тренажер.

    2) Примеры со скобками в пределах до 100. Онлайн тренажер.

    3) Примеры со скобками. Тренажер №2

    4) Вставь пропущенное число — примеры со скобками. Тренажер

    2 Примеры со скобками (сложение, вычитание, умножение, деление)

    Теперь рассмотрим примеры, в которых кроме сложения и вычитания есть умножение и деление.

    Сначала рассмотрим примеры без скобок:

    Есть одна хитрость, как не запутаться при решении примеров на порядок действий. Если нет скобок, то выполняем действия умножения и деления, далее переписываем пример, записывая вместо этих действий полученные результаты. Затем выполняем сложение и вычитание по порядку:

    Если в примере есть скобки, то сначала нужно избавиться от скобок: переписать пример, записывая вместо скобок полученный в них результат. Затем нужно выделить мысленно части примера, разделенные знаками «+» и «-«, и посчитать каждую часть отдельно. Затем выполнить сложение и вычитание по порядку:

    3 Примеры, в которых много действий

    Если в примере много действий, то удобнее будет не расставлять порядок действий во всем примере, а выделить блоки, и решить каждый блок отдельно. Для этого находим свободные знаки «+» и «–» (свободные — значит не в скобках, на рисунке показаны стрелочками).

    Эти знаки и будут делить наш пример на блоки:

    Выполняя действия в каждом блоке не забываем про порядок действий, приведенный выше в статье. Решив каждый блок, выполняем действия сложения и вычитания по порядку.

    А теперь закрепляем решение примеров на порядок действий на тренажерах!

    Если у вас не открываются игры или тренажёры, читайте .

    Когда мы работаем с различными выражениями, включающими в себя цифры, буквы и переменные, нам приходится выполнять большое количество арифметических действий. Когда мы делаем преобразование или вычисляем значение, очень важно соблюдать правильную очередность этих действий. Иначе говоря, арифметические действия имеют свой особый порядок выполнения.

    Yandex.RTB R-A-339285-1

    В этой статье мы расскажем, какие действия надо делать в первую очередь, а какие после. Для начала разберем несколько простых выражений, в которых есть только переменные или числовые значения, а также знаки деления, умножения, вычитания и сложения. Потом возьмем примеры со скобками и рассмотрим, в каком порядке следует вычислять их. В третьей части мы приведем нужный порядок преобразований и вычислений в тех примерах, которые включают в себя знаки корней, степеней и других функций.

    Определение 1

    В случае выражений без скобок порядок действий определяется однозначно:

    1. Все действия выполняются слева направо.
    2. В первую очередь мы выполняем деление и умножение, во вторую – вычитание и сложение.

    Смысл этих правил легко уяснить. Традиционный порядок записи слева направо определяет основную последовательность вычислений, а необходимость сначала умножить или разделить объясняется самой сутью этих операций.

    Возьмем для наглядности несколько задач. Мы использовали только самые простые числовые выражения, чтобы все вычисления можно было провести в уме. Так можно быстрее запомнить нужный порядок и быстро проверить результаты.

    Пример 1

    Условие: вычислите, сколько будет 7 − 3 + 6 .

    Решение

    В нашем выражении скобок нет, умножение и деление также отсутствуют, поэтому выполняем все действия в указанном порядке. Сначала вычитаем три из семи, затем прибавляем к остатку шесть и в итоге получаем десять. Вот запись всего решения:

    7 − 3 + 6 = 4 + 6 = 10

    Ответ: 7 − 3 + 6 = 10 .

    Пример 2

    Условие: в каком порядке нужно выполнять вычисления в выражении 6: 2 · 8: 3 ?

    Решение

    Чтобы дать ответ на этот вопрос, перечитаем правило для выражений без скобок, сформулированное нами до этого. У нас здесь есть только умножение и деление, значит, мы сохраняем записанный порядок вычислений и считаем последовательно слева направо.

    Ответ: сначала выполняем деление шести на два, результат умножаем на восемь и получившееся в итоге число делим на три.

    Пример 3

    Условие: подсчитайте, сколько будет 17 − 5 · 6: 3 − 2 + 4: 2 .

    Решение

    Сначала определим верный порядок действий, поскольку у нас здесь есть все основные виды арифметических операций – сложение, вычитание, умножение, деление. Первым делом нам надо разделить и умножить. Эти действия не имеют приоритета друг перед другом, поэтому выполняем их в написанном порядке справа налево. То есть 5 надо умножить на 6 и получить 30 , потом 30 разделить на 3 и получить 10 . После этого делим 4 на 2 , это 2 . Подставим найденные значения в исходное выражение:

    17 − 5 · 6: 3 − 2 + 4: 2 = 17 − 10 − 2 + 2

    Здесь уже нет ни деления, ни умножения, поэтому делаем оставшиеся вычисления по порядку и получаем ответ:

    17 − 10 − 2 + 2 = 7 − 2 + 2 = 5 + 2 = 7

    Ответ: 17 − 5 · 6: 3 − 2 + 4: 2 = 7 .

    Пока порядок выполнения действий не заучен твердо, можно ставить над знаками арифметических действий цифры, означающие порядок вычисления. Например, для задачи выше мы могли бы записать так:

    Если у нас есть буквенные выражения, то с ними мы поступаем точно так же: сначала умножаем и делим, затем складываем и вычитаем.

    Что такое действия первой и второй ступени

    Иногда в справочниках все арифметические действия делят на действия первой и второй ступени. Сформулируем нужное определение.

    К действиям первой ступени относятся вычитание и сложение, второй – умножение и деление.

    Зная эти названия, мы можем записать данное ранее правило относительно порядка действий так:

    Определение 2

    В выражении, в котором нет скобок, сначала надо выполнить действия второй ступени в направлении слева направо, затем действия первой ступени (в том же направлении).

    Порядок вычислений в выражениях со скобками

    Скобки сами по себе являются знаком, который сообщает нам нужный порядок выполнения действий. В таком случае нужное правило можно записать так:

    Определение 3

    Если в выражении есть скобки, то первым делом выполняется действие в них, после чего мы умножаем и делим, а затем складываем и вычитаем по направлению слева направо.

    Что касается самого выражения в скобках, его можно рассматривать в качестве составной части основного выражения. При подсчете значения выражения в скобках мы сохраняем все тот же известный нам порядок действий. Проиллюстрируем нашу мысль примером.

    Пример 4

    Условие: вычислите, сколько будет 5 + (7 − 2 · 3) · (6 − 4) : 2 .

    Решение

    В данном выражении есть скобки, поэтому начнем с них. Первым делом вычислим, сколько будет 7 − 2 · 3 . Здесь нам надо умножить 2 на 3 и вычесть результат из 7:

    7 − 2 · 3 = 7 − 6 = 1

    Считаем результат во вторых скобках. Там у нас всего одно действие: 6 − 4 = 2 .

    Теперь нам нужно подставить получившиеся значения в первоначальное выражение:

    5 + (7 − 2 · 3) · (6 − 4) : 2 = 5 + 1 · 2: 2

    Начнем с умножения и деления, потом выполним вычитание и получим:

    5 + 1 · 2: 2 = 5 + 2: 2 = 5 + 1 = 6

    На этом вычисления можно закончить.

    Ответ: 5 + (7 − 2 · 3) · (6 − 4) : 2 = 6 .

    Не пугайтесь, если в условии у нас содержится выражение, в котором одни скобки заключают в себе другие. Нам надо только применять правило выше последовательно по отношению ко всем выражениям в скобках. Возьмем такую задачу.

    Пример 5

    Условие: вычислите, сколько будет 4 + (3 + 1 + 4 · (2 + 3)) .

    Решение

    У нас есть скобки в скобках. Начинаем с 3 + 1 + 4 · (2 + 3) , а именно с 2 + 3 . Это будет 5 . Значение надо будет подставить в выражение и подсчитать, что 3 + 1 + 4 · 5 . Мы помним, что сначала надо умножить, а потом сложить: 3 + 1 + 4 · 5 = 3 + 1 + 20 = 24 . Подставив найденные значения в исходное выражение, вычислим ответ: 4 + 24 = 28 .

    Ответ: 4 + (3 + 1 + 4 · (2 + 3)) = 28 .

    Иначе говоря, при вычислении значения выражения, включающего скобки в скобках, мы начинаем с внутренних скобок и продвигаемся к внешним.

    Допустим, нам надо найти, сколько будет (4 + (4 + (4 − 6: 2)) − 1) − 1 . Начинаем с выражения во внутренних скобках. Поскольку 4 − 6: 2 = 4 − 3 = 1 , исходное выражение можно записать как (4 + (4 + 1) − 1) − 1 . Снова обращаемся к внутренним скобкам: 4 + 1 = 5 . Мы пришли к выражению (4 + 5 − 1) − 1 . Считаем 4 + 5 − 1 = 8 и в итоге получаем разность 8 — 1 , результатом которой будет 7 .

    Порядок вычисления в выражениях со степенями, корнями, логарифмами и иными функциями

    Если у нас в условии стоит выражение со степенью, корнем, логарифмом или тригонометрической функцией (синусом, косинусом, тангенсом и котангенсом) или иными функциями, то первым делом мы вычисляем значение функции. После этого мы действуем по правилам, указанным в предыдущих пунктах. Иначе говоря, функции по степени важности приравниваются к выражению, заключенному в скобки.

    Разберем пример такого вычисления.

    Пример 6

    Условие: найдите, сколько будет (3 + 1) · 2 + 6 2: 3 − 7 .

    Решение

    У нас есть выражение со степенью, значение которого надо найти в первую очередь. Считаем: 6 2 = 36 . Теперь подставим результат в выражение, после чего оно примет вид (3 + 1) · 2 + 36: 3 − 7 .

    (3 + 1) · 2 + 36: 3 − 7 = 4 · 2 + 36: 3 − 7 = 8 + 12 − 7 = 13

    Ответ: (3 + 1) · 2 + 6 2: 3 − 7 = 13 .

    В отдельной статье, посвященной вычислению значений выражений, мы приводим и другие, более сложные примеры подсчетов в случае выражений с корнями, степенью и др. Рекомендуем вам с ней ознакомиться.

    Если вы заметили ошибку в тексте, пожалуйста, выделите её и нажмите Ctrl+Enter

    И вычислении значений выражений действия выполняются в определенной очередности, иными словами, нужно соблюдать порядок выполнения действий .

    В этой статье мы разберемся, какие действия следует выполнять сначала, а какие следом за ними. Начнем с самых простых случаев, когда выражение содержит лишь числа или переменные, соединенные знаками плюс, минус, умножить и разделить. Дальше разъясним, какого порядка выполнения действий следует придерживаться в выражениях со скобками. Наконец, рассмотрим, в какой последовательности выполняются действия в выражениях, содержащих степени, корни и другие функции.

    Навигация по странице.

    Сначала умножение и деление, затем сложение и вычитание

    В школе дается следующее правило, определяющее порядок выполнения действий в выражениях без скобок :

    • действия выполняются по порядку слева направо,
    • причем сначала выполняется умножение и деление, а затем – сложение и вычитание.

    Озвученное правило воспринимается достаточно естественно. Выполнение действий по порядку слева направо объясняется тем, что у нас принято вести записи слева направо. А то, что умножение и деление выполняется перед сложением и вычитанием объясняется смыслом, который в себе несут эти действия.

    Рассмотрим несколько примеров применения этого правила. Для примеров будем брать простейшие числовые выражения, чтобы не отвлекаться на вычисления, а сосредоточиться именно на порядке выполнения действий.

    Пример.

    Выполните действия 7−3+6 .

    Решение.

    Исходное выражение не содержит скобок, а также оно не содержит умножения и деления. Поэтому нам следует выполнить все действия по порядку слева направо, то есть, сначала мы от 7 отнимаем 3 , получаем 4 , после чего к полученной разности 4 прибавляем 6 , получаем 10 .

    Кратко решение можно записать так: 7−3+6=4+6=10 .

    Ответ:

    7−3+6=10 .

    Пример.

    Укажите порядок выполнения действий в выражении 6:2·8:3 .

    Решение.

    Чтобы ответить на вопрос задачи, обратимся к правилу, указывающему порядок выполнения действий в выражениях без скобок. В исходном выражении содержатся лишь действия умножения и деления, а согласно правилу, их нужно выполнять по порядку слева направо.

    Ответ:

    Сначала 6 делим на 2 , это частное умножаем на 8 , наконец, полученный результат делим на 3.

    Пример.

    Вычислите значение выражения 17−5·6:3−2+4:2 .

    Решение.

    Сначала определим, в каком порядке следует выполнять действия в исходном выражении. Оно содержит и умножение с делением, и сложение с вычитанием. Сначала слева направо нужно выполнить умножение и деление. Так 5 умножаем на 6 , получаем 30 , это число делим на 3 , получаем 10 . Теперь 4 делим на 2 , получаем 2 . Подставляем в исходное выражение вместо 5·6:3 найденное значение 10 , а вместо 4:2 — значение 2 , имеем 17−5·6:3−2+4:2=17−10−2+2 .

    В полученном выражении уже нет умножения и деления, поэтому остается по порядку слева направо выполнить оставшиеся действия: 17−10−2+2=7−2+2=5+2=7 .

    Ответ:

    17−5·6:3−2+4:2=7 .

    На первых порах, чтобы не перепутать порядок выполнения действий при вычислении значения выражения, удобно над знаками действий расставить цифры, соответствующие порядку их выполнения. Для предыдущего примера это выглядело бы так: .

    Этого же порядка выполнения действий – сначала умножение и деление, затем сложение и вычитание — следует придерживаться и при работе с буквенными выражениями.

    Действия первой и второй ступени

    В некоторых учебниках по математике встречается разделение арифметических действий на действия первой и второй ступени. Разберемся с этим.

    Определение.

    Действиями первой ступени называют сложение и вычитание, а умножение и деление называют действиями второй ступени .

    В этих терминах правило из предыдущего пункта, определяющее порядок выполнения действий, запишется так: если выражение не содержит скобок, то по порядку слева направо сначала выполняются действия второй ступени (умножение и деление), затем – действия первой ступени (сложение и вычитание).

    Порядок выполнения арифметических действий в выражениях со скобками

    Выражения часто содержат скобки, указывающие порядок выполнения действий . В этом случае правило, задающее порядок выполнения действий в выражениях со скобками , формулируется так: сначала выполняются действия в скобках, при этом также по порядку слева направо выполняется умножение и деление, затем – сложение и вычитание.

    Итак, выражения в скобках рассматриваются как составные части исходного выражения, и в них сохраняется уже известный нам порядок выполнения действий. Рассмотрим решения примеров для большей ясности.

    Пример.

    Выполните указанные действия 5+(7−2·3)·(6−4):2 .

    Решение.

    Выражение содержит скобки, поэтому сначала выполним действия в выражениях, заключенных в эти скобки. Начнем с выражения 7−2·3 . В нем нужно сначала выполнить умножение, и только потом вычитание, имеем 7−2·3=7−6=1 . Переходим ко второму выражению в скобках 6−4 . Здесь лишь одно действие – вычитание, выполняем его 6−4=2 .

    Подставляем полученные значения в исходное выражение: 5+(7−2·3)·(6−4):2=5+1·2:2 . В полученном выражении сначала выполняем слева направо умножение и деление, затем – вычитание, получаем 5+1·2:2=5+2:2=5+1=6 . На этом все действия выполнены, мы придерживались такого порядка их выполнения: 5+(7−2·3)·(6−4):2 .

    Запишем краткое решение: 5+(7−2·3)·(6−4):2=5+1·2:2=5+1=6 .

    Ответ:

    5+(7−2·3)·(6−4):2=6 .

    Бывает, что выражение содержит скобки в скобках. Этого бояться не стоит, нужно лишь последовательно применять озвученное правило выполнения действий в выражениях со скобками. Покажем решение примера.

    Пример.

    Выполните действия в выражении 4+(3+1+4·(2+3)) .

    Решение.

    Это выражение со скобками, это означает, что выполнение действий нужно начинать с выражения в скобках, то есть, с 3+1+4·(2+3) . Это выражение также содержит скобки, поэтому нужно сначала выполнить действия в них. Сделаем это: 2+3=5 . Подставив найденное значение, получаем 3+1+4·5 . В этом выражении сначала выполняем умножение, затем – сложение, имеем 3+1+4·5=3+1+20=24 . Исходное значение, после подстановки этого значения, принимает вид 4+24 , и остается лишь закончить выполнение действий: 4+24=28 .

    Ответ:

    4+(3+1+4·(2+3))=28 .

    Вообще, когда в выражении присутствуют скобки в скобках, то часто бывает удобно выполнение действий начинать с внутренних скобок и продвигаться к внешним.

    Например, пусть нам нужно выполнить действия в выражении (4+(4+(4−6:2))−1)−1 . Сначала выполняем действия во внутренних скобках, так как 4−6:2=4−3=1 , то после этого исходное выражение примет вид (4+(4+1)−1)−1 . Опять выполняем действие во внутренних скобках, так как 4+1=5 , то приходим к следующему выражению (4+5−1)−1 . Опять выполняем действия в скобках: 4+5−1=8 , при этом приходим к разности 8−1 , которая равна 7 .

    Числовые и буквенные выражения могут содержать знаки различных арифметических действий. При преобразовании выражений и вычислении значений выражений действия выполняются в определенной очередности, так как существует строгий порядок выполнения математических действий

    Сначала умножение и деление, затем сложение и вычитание


    Порядок выполения действий в выражениях без скобок:

    — действия выполняются по порядку слева направо,

    — причем сначала выполняется умножение и деление, а затем – сложение и вычитание .

    1. Рассмотрим пример: выполните действия 17−3+6

    Исходное выражение не содержит умножения и деления и не содержит скобок. Поэтому нам следует выполнить все действия по порядку слева направо , то есть, сначала мы от 17 отнимаем 3, получаем 14, после чего к полученной разности 14 прибавляем 6, получаем 20.

    Кратко решение можно записать так: 17 − 3 + 6 = 14 + 6 = 20

    2. Вычислите значение выражения 17 − 5 · 6: 3 − 2 + 4: 2

    Сначала определим, в каком порядке следует выполнять действия в выражении. Оно содержит и умножение с делением, и сложение с вычитанием. Сначала слева направо нужно выполнить умножение и деление .

    4: 2 теперь 4 делим на 2, получаем 2.

    Подставляем в исходное выражение вместо 5 · 6: 3 найденное значение 10, а вместо 4: 2 — значение 2, получаем следующее выражение 17 − 5 · 6: 3 − 2 + 4: 2 = 17 − 10 − 2+ 2 .

    В полученном выражении уже нет умножения и деления, поэтому остается по порядку слева направо выполнить оставшиеся действия: 17 − 10 − 2 + 2 = 7 − 2 + 2 = 5 + 2 = 7.

    Действия первой и второй ступени


    Для удобства принятия решения о последовательности выполнения действий их разделили на две ступени:

    первая ступень — сложение и вычитание,

    вторая ступень — умножение и деление.

    Если выражение не содержит скобок, то по порядку слева направо сначала выполняются действия второй ступени (умножение и деление), затем – действия первой ступени (сложение и вычитание)

    Порядок выполнения арифметических действий в выражениях со скобками

    Правило, задающее порядок выполнения действий в выражениях со скобками, формулируется так: сначала выполняются действия в скобках, при этом также по порядку слева направо выполняется умножение и деление, затем – сложение и вычитание.

    Рассмотрим пример: 99: (45 – 39 + 5) – 25: 5

    Порядок вычисления такой. Сначала выполним действия в скобках:

    45 – 39 = 6 ; 6 + 5 = 11 ,

    затем действия второй ступени

    Заказ операций — Pemdas, Bodmas, DMAS

    Некоторые основные математические терминологии:

    + представляет сумму или добавление

    представляет разность или вычитание

    × или Представляют продукт или множество

    × или . ÷  или / представляет деление

    []  или  ()  представляет скобки или круглые скобки

    Будьте осторожны  вместо  до  путайте  (круглые скобки) или [квадратные скобки] с |столбиками абсолютных значений|. Это не одни и те же символы, и для их оценки применяются разные правила. Мы обсудим |столбики абсолютных значений| позже.

    Математические «Операции» средние вещи, такие как , добавить , Вычитание , Умножение , Разделите , Квадрат и т. Д. (2 × 5 2  – 3 ÷ 6 × 14)

    Какую часть следует вычислить в первую очередь?

    Итак, давным-давно люди согласились следовать некоторым правилам при выполнении вычислений, а именно:

    Делать что-то в P арентезы Первые

    4 × (5 + 3) = 4 × 8 = 32 (справа)

    4 × (5 + 3) = 20 + 3 = 23 (неверно)

    E умножить на степени (степени, корни) перед умножением, делением, сложением или вычитанием

    5 × 2 2  = 5 × 4 = 20 (справа)

    5 × 2 2 = 10 2 = 100 (неправильно)

    M Ultiply или D IVIDE Перед вашим A DD или S UBTRACT 15 = 17 (правильно)

    2 + 5 × 3 = 7 × 3 = 21 (неверно)

    Как мне все это запомнить… ? ПЕМДАС !

    33

    MD
    P Первые скобки
    E Показатель степени
    Умножение и деление (оба имеют одинаковый ранг, и если они сойдутся, то решать слева направо)
    30 ÷ 5 × 3 = 6 × 3 = 18 (справа)
    30 ÷ 5 × 3 = 30 ÷ 15 = 2 (неправильно)
    AS Дополнение и вычитание
    В британских Subtract), а в Канаде говорят BEDMAS (скобки, экспоненты, деление, умножение, сложение, вычитание). Он также известен как DMAS (разделяй, умножай, добавляй, вычитай). Все это означает одно и то же!

    Просто помните, что хотя умножение стоит перед делением в этой аббревиатуре, они имеют одинаковый ранг. PEMDAS говорит нам, что «умножение и деление» должны быть выполнены до «сложения и вычитания».

    Например,

    35 ÷ 5 + 2 × 6 равен

    (35 ÷ 5) + (2 × 6) = 19

    и не равен

    35 ÷ (5. + 2) × 6 = 35 ÷ 7 × 6

    Когда у вас есть куча операций одного ранга (Умножение и Деление) ,  вы просто работаете слева направо. Например,

    35 ÷ 7 × 6

    не

    35 ÷ (7 × 6) = 35 ÷ 42,

    , но это скорее

    (35 ÷ 7) × 60002 (35 ÷ 7) = 5 × 6 = 30,

    , потому что, двигаясь слева направо, вы сначала доберетесь до знака деления.

    Вернемся к последнему рассмотренному нами примеру:

    35 ÷ 7 × 6 = 30

    В этой задаче мы решали в порядке появления, то есть слева направо. Сначала мы разделили 35 на 7, в результате чего получилось 5. Затем мы умножили 5 на 6, чтобы получить 30.

    Мы можем повторить этот вопрос, превратив знак деления ( ÷ ) в знак умножения (×).

    Это можно сделать, перевернув число после знака деления вверх ногами и изменив символ деления на символ умножения.

    В данном случае мы перевернули 7 вверх ногами. Теперь мы можем просто выразить выражение, чтобы получить тот же результат, что и раньше.

    Будьте осторожны. Иногда можно совершить ошибку, перевернув все после символа деления.

    Это было бы правильно, если бы была скобка, например 35 ÷ (7 × 6). Тогда это стало бы

    7 × 9 + (2 × 5 2  – 3 ÷ 6 × 14)

    Подытожим:

    • Умножение и деление 3 предшествуют сложению2 и34вычитанию 90 имеют одинаковый ранг
    • Если умножение и деление объединяются
      • , то решайте слева направо или
      • замените деление на умножение и переверните число, которое стоит после деления

    Как и у большинства людей, сдающих GRE, ваши знания по математике могут быть немного заржаветыми. Хорошей новостью является то, что GRE проверяет вас только по математике, которую вы уже выучили в старшей школе. Если вы беспокоитесь, что забыли большую часть того, что выучили в старшей школе, вам придется…

    читать дальше

     Стандартное отклонение – важный статистический термин, проверенный на GRE. Это дает вам представление об отклонении или разбросе набора чисел от его среднего значения; следовательно, низкое стандартное отклонение означает, что числа очень близки к среднему, и наоборот…

    подробнее

    Каждое целое число больше 1 является либо простым, либо составным числом. Все составные числа можно представить в виде произведения простых чисел. Например, 6 можно выразить как 2 × 3. Простые делители 6 — это 2 и 3. Тогда как выражение 2 × 3 называется…

    читать далее

    Давайте посмотрим, как мы используем диаграмму, чтобы ответить на этот вопрос: сколько положительных факторов имеет 100? Настройте диаграмму: 100 Левая колонка Правая колонка 1 100 2 50 4 25 5 20 10 10 Помните правило: как только факторы повторяются (например, 10 и 10), остановитесь. С…

    подробнее

    Вы уже видели прекрасную таблицу множителей — простой способ найти все множители целого числа. Давайте посмотрим, как мы используем диаграмму, чтобы ответить на этот вопрос: сколько положительных факторов имеет число 140? Вопрос касается факторов, поэтому таблица факторов…

    читать дальше

    Некоторые целочисленные вопросы могут потребовать от вас найти множители целого числа. Таблица множителей — это основной метод нахождения всех множителей любого целого числа. Этот метод также может быть полезен для вопросов о том, сколько множителей имеет конкретное целое число. Техника…

    подробнее

    Простые числа играют центральную роль в целочисленных вопросах. Излишняя самоуверенность здесь опасна: в то время как почти каждый может без труда назвать определение простого числа, поле на самом деле изобилует неправильными представлениями. Мы здесь, чтобы убедиться, что вы знаете все…

    читать дальше

    Сформулируем правила сложения, вычитания, умножения и деления четных и нечетных чисел. Вот правила сложения/вычитания четных и нечетных чисел: Четные ± четные = четные (например, 2 + 2 = 4; -4 — 2 = -6) Нечетные ± нечетные = четные (например, 1 + 1 = 2; -31 — 1 = -32)…

    подробнее

    Начнем с основных определений: Четное: любое целое число, которое делится на 2. Примеры: 2, 4, 14. Нечетное: любое целое число, которое не делится на 2, т. е. дает остаток 1 при делении на 2. Примеры: 1, 3, 5, 7, 9 Пока все хорошо. Рассмотрите этот потенциал…

    читать дальше

    Найдите остаток от деления 5 142 376 298 на 9? Метод длинного деления точно скажет вам остаток от деления 5 142 376,29.8 на 9, но вы можете себе представить, сколько времени это займет. Хорошая новость заключается в том, что, как и делимость. ..

    читать дальше

    7.2: Алгоритмы деления — Математика LibreTexts

    1. Последнее обновление
    2. Сохранить как PDF
  • Идентификатор страницы
    70324
    • Джули Харланд
    • Колледж МираКоста

    Как мы видели, деление можно рассматривать как противоположность умножения.

    Другой способ представить это как ПОВТОРНОЕ ВЫЧИТАНИЕ .

    Давайте сделаем DIVISION простым способом!

    Верно. Деление на самом деле просто «повторяющееся вычитание». Если вас попросят разделить 17 на 3, что записывается как \(17 \div 3\), вам нужно выяснить, сколько троек можно вычесть из 17. Вероятно, вы уже знаете, что ответ — 5 r. 2, но давайте посмотрим, как это работает, посчитав, сколько раз можно вычесть 3 из 17. Посмотрите на работу ниже. Показаны три разных способа. Вы делаете эшафот (выглядит как палач) и следите за тем, сколько троек вы вычитаете на каждом шагу справа от эшафота. В первом 3 вычитаются по одному, и всего было вычтено пять троек. Во втором сначала вычитаются две тройки, затем еще 1, затем еще 2, всего вычитается пять троек. В третьем вычитаются четыре тройки, а затем еще 1, и снова вычитается пять троек. Каждый раз, когда вы вычитаете, смотрите, какое число осталось. Если это 3 или более, вы можете вычесть еще несколько троек. Но когда вы получаете число меньше 3, вы знаете, что 3 больше нельзя вычесть, так что это ваш остаток. Для этой задачи 2 было остатком. Вам нужно посчитать, сколько троек вы вычли. Справа от эшафота пишешь сколько 3 вы вычитаете по ходу дела. Просто сложите эти числа, чтобы узнать, сколько троек вы вычли. Ответ на задачу: сколько троек вы вычли, а потом нужно еще написать остаток. В этом случае \(17 \div 3 = 5\) r. 2. Всегда проверяйте свой ответ, умножая делитель (3) на частное (5) и добавляя остаток (2): \(3 \times 5 + 2 = 15 + 2 = 17\). Это проверяет!

    Деление в длинное с большими числами часто бывает затруднено, потому что приходится умножать в уме, а ошибиться легко. Но, используя этот новый метод многократного вычитания, наряду с частичным умножением, которое вы делаете заранее, используя многократное сложение, вам никогда не придется умножать в уме (или умножать вообще), и вам никогда не придется угадывать, сколько раз одно число «переходит» в другое число. Этот метод требует немного больше бумаги, но упрощает задачу и практически надежен. Многим людям не нравится выполнять деление в длинное число, потому что трудно всегда угадывать точное правильное число, а иногда у них начинает болеть голова от того, что они слишком много думают. Мы собираемся устранить эту проблему. Важно то, что вы знаете, что означает деление (повторное вычитание), вы знаете, как получить правильный ответ, используя метод, который имеет для вас смысл, и вы знаете, как проверить свой ответ, чтобы убедиться, что он правильный.

    Итак, допустим, вам нужно было решить эту задачу на деление: \(361 \div 53\).

    ЮК! Это не похоже на какое-то развлечение. На самом деле, все, что спрашивается, это сколько 53 можно вычесть из 361. Мы можем вычитать 53 по одному, но это может занять много времени. Мы можем решить вычесть более одного 53 за раз.

    Прежде чем приступить к делению, давайте быстро составим таблицу частичного умножения на 53, используя ПОВТОРНОЕ СЛОЖЕНИЕ! Эта таблица — ваша подготовительная работа к делению. Простой способ составить частичную таблицу умножения на 53 без умножения влечет за собой удвоение чисел. Мы знаем, что \(1 \times 53 = 53\). Чтобы вычислить \(2 \times 53\), просто сложите два числа 53 вместе. Два числа 53, добавленные к еще двум числам 53, будут равны четырем числам 53, или \(4 \x 53\). Четыре 53, добавленные к еще четырем 53, будут равны восьми 53, или \(8 \х53\). Хорошо, а как ты это покажешь? Начните с одного 53 и удвойте его, чтобы найти два 53. Удвойте это число, чтобы найти четыре 53-х. Удвойте это число, чтобы найти восемь 53-х. Посмотрите в левую часть окна, показанного ниже. Посмотрите, как 1 указывает на 53, что означает \(1 \times 53\). Если вы удвоите 53 (53 + 53), то теперь вы знаете \(2 \times 53 = 106\), поэтому 2 указывает на 106. Если вы удвоите это (106 + 106), вы теперь знаете \(4 \times 53 = 212\), поэтому 4 указывает на 212. И если вы удвоите это число (212 + 212), то теперь вы знаете \(8 \times 53 = 424\), поэтому 8 указывает на 424. Разве Разве это не простой способ найти \(8 \times 53\)? Но самое интересное, что вы также знаете \(1 \times 53, 2 \times 53, 4 \times 53,\) И \(8 \times 53\)! Это полезно знать, чтобы выполнить деление в длину. Вместо того, чтобы гадать, сколько раз 53 «входит» в 361, мы отмечаем, сколько 53 можно вычесть из 361 (недооценка допустима). Умножать в уме тоже не нужно! В рамке справа показано, как использовать многократное вычитание для решения задачи на деление: \(361 \div 53\). Сначала составим частичную таблицу умножения. Затем напишите задачу на деление, используя подмости. Посмотрите на таблицу умножения и выберите наибольшее число, которое можно вычесть из 361, то есть 212. Вычтите 212 из 361 и запишите справа 4, потому что вы только что вычли 4 53. Самое большое число, которое можно вычесть из 149будет 106. Если вы сделаете это, вы вычтете еще 2 53-х, поэтому напишите 2 справа. После вычитания остается 43, и больше 53-х вычесть нельзя. Следовательно, 43 — это остаток. Сложите числа справа, чтобы увидеть, сколько 53 вы вычли. Ответ 6 р. 43. Для проверки умножьте \(53 \times 6\) и прибавьте 43. Ответ: 361.

    П.С. Вы можете умножить 53 на 6, добавив 106 + 106 + 106, так как это два 53, сложенные вместе 3 раза, что составляет шесть 53!!!

    Вот еще несколько примеров для изучения. Сначала составляется неполная таблица умножения. Эта подготовительная работа, которая включает в себя несколько дополнений, значительно упрощает деление. Затем задача решается с помощью многократного вычитания (с помощью частичной таблицы умножения) с использованием лески. После сложения, сколько раз делитель был вычтен из делимого (числа справа от матрицы), ответ (частное и остаток) записывается в рамке над задачей. Наконец, чек показан. Также проверьте, чтобы остаток был меньше делителя.

    В примерах до сих пор все частные были меньше 10. Можно продолжать удваивать делитель в частичной таблице умножения, так что вы можете найти делитель, умноженный на любую степень 2 — 1, 2, 4, 8, 16, 32, 64 и т. д. Вы перестанете удваивать, если заметите, что еще одно удвоение дает число больше частного. Фактически, в первом примере на предыдущей странице, \(279 \div 37\), мне действительно не нужно было удваивать 148, чтобы вычислить \(8 \times 37\), так как удвоение 148 дает мне большее число чем 279. В третьем примере, \(887 \div 231\), мне не нужно было удваивать 462, чтобы вычислить \(4 \times 231\), так как удвоение 462 дает число больше 887. Задачи можно было сократить. немного если бы я заметил, что раньше времени. Они будут выглядеть так (галочка опущена).

    Хорошо, теперь, когда вы изучили несколько примеров, пришло время попробовать использовать этот метод. Первые четыре задачи точно такие же, как и в предыдущих примерах. Если вы застряли, оглянитесь на них и скопируйте, как я их сделал, на другой лист бумаги. Затем повторите попытку.

    Упражнение 1

    Для каждой задачи на деление выполните подготовительную работу, используя метод удвоения (удваивайте столько, сколько необходимо), чтобы сначала составить частичную таблицу умножения. Затем нарисуйте каркас и используйте многократное вычитание для каждого деления. Когда закончите, убедитесь, что остаток меньше делителя. Затем проверьте ответ. Все работы должны быть показаны в отведенном для этого месте. После проверки напишите ответ на отведенном месте в начале задачи.

    а. \(361 \дел 53\) = _______________

    Частичная таблица умножения: Леса: Чек:

    б. \(279 \дел 37\) = _______________

    Частичная таблица умножения: Леса: Чек:

    в. \(200 \дел 24\) = ___________________

    Частичная таблица умножения: Леса: Чек:

    д. \(887 \дел 231\) = ______________

    Частичная таблица умножения: Леса: Чек:

    эл. \(415 \дел 72\) = _______________

    Частичная таблица умножения: Леса: Чек:

    ф. \(1235 \дел 214\) = _____________

    Частичная таблица умножения: Леса: Чек:

    г. \(3128 \дел 321\) = _____________

    Частичная таблица умножения: Леса: Чек:

    Следующий пример показывает, что удвоение может продолжаться таким образом. Вы перестаете удваивать, если еще одно удвоение дает число больше, чем частное. Ниже находится \(1230 \div 27\).

    Я благодарю и доверяю моему сыну Якобу за этот метод удвоения, позволяющий вычислить более чем восьмикратный делитель. Я всегда использовал метод, использующий разрядные значения и числа, кратные 10, к которым мы вернемся после того, как проделаем несколько подобных операций. Якоб, девятилетний ученик четвертого класса, ненавидел длинное деление, потому что оно не имело для него смысла, и у него были проблемы с «угадыванием». Прежде чем я смог показать ему свой метод, в котором вы используете метод удвоения, чтобы составить частичную таблицу умножения только до 8-кратного делителя, он решил все задачи, как показано выше. Наконец-то это стало ему понятно, и он все сделал правильно!!! Это открыло для меня совершенно новый способ мышления. Дети и студенты могут быть великими новаторами! Замечательно, когда у вас есть возможность исследовать и открывать для себя что-то новое и найти собственное решение, которое имеет смысл. Мы должны помнить, что нет ничего святого ни в одном алгоритме деления, ни в любой другой операции.

    Хорошо, пришло время попробовать еще несколько задач, используя метод удвоения, где частное больше 10. Первая задача точно такая же, как и выше. Если вы застряли, оглянитесь на него и скопируйте, как я это сделал, на другой лист бумаги. Затем повторите попытку.

    Упражнение 2

    Для каждой задачи на деление (начиная со следующей страницы) используйте метод удвоения (удваивайте столько, сколько необходимо), чтобы составить частичную таблицу умножения. Затем нарисуйте каркас и используйте многократное вычитание для каждого деления. Затем проверьте ответ и убедитесь, что остаток меньше делимого. Все работы должны быть показаны. После проверки напишите ответ на отведенном месте в начале задачи.

    Проверка: \(\begin{align} 27 \\ \underline{\times 45} \\ 1215 \\ \underline{+ 15} \\ 1230 \end{aligned}\)

    Калькулятор может быть использован для проверьте ответ. Например, проверить, что 45 р. 15 — это правильный ответ на \(1230 \div 27\), вам нужно умножить 27 на 45 и прибавить 15. Ответ должен быть 1230. Вы можете использовать калькулятор, чтобы выполнить умножение и сложение, но запишите шаги. , Это будет выглядеть так, как показано справа.

    а. \(1230 \дел 27\) = ______________

    Частичная таблица умножения: Леса: Чек:

    б. \(603 \дел 48\) = _______________

    Частичная таблица умножения: Леса: Чек:

    в. \(1346 \дел 41\) = ______________

    Частичная таблица умножения: Леса: Чек:

    д. \(3512 \дел 43\) = ______________

    Частичная таблица умножения: Леса: Чек:

    Возможно, вы заметили, что неполная таблица умножения может стать длинной, если вам придется продолжать удваивать некоторое время. Кроме того, не так просто сложить числа справа от эшафота. Другой подход заключается в использовании того факта, что при умножении целого числа на 10 или 100 и т. д. просто добавляются нули в конце числа. Например, если вы знаете, что 53 умножить на 2 — это 106, то 53 умножить на 20 — это 1060, а 53 умножить на 200 — это 10600. Мы будем использовать этот факт для деления с помощью повторного вычитания, когда частное больше 10.

    Вспомните последнюю задачу, которую вы решили в упражнении 2: \(3512 \div 43\). Вам приходилось удваивать, пока вы не выяснили, сколько будет 64 умножить на 43. В поле внизу этой страницы указан другой способ решения той же задачи. Объяснение и шаги находятся на этой и следующей странице.

    Шаг 1: Используйте удвоение для составления частичной таблицы умножения до 8 раз 43.

    Шаг 2: Нарисуйте леса.

    Шаг 3: Сканируйте 3512, начиная с крайней левой цифры, пока не получите число, большее или равное 43. Попробуйте 3, затем 35, затем 351. Обратите внимание, сколько еще цифр после 351. В этом случае есть только один (2). Это означает, что за один раз вы можете убрать число, кратное десяти, умноженное на 43. (В старом стандартном алгоритме, когда вы угадываете и ставите число выше 1 в 3512, вы ставите его в разряд десятков.) Итак, мы ставим ноль под дополнительными числами в делимом (2) и то же самое. количество нулей справа от эшафота. Оставьте место для цифры перед 0 справа от каркаса.

    Шаг 4: Теперь взгляните на неполную таблицу умножения, чтобы определить наибольшее число, которое можно вычесть из 351. Это 344, то есть 8 умножить на 43. Обратите внимание, что если 8 умножить на 43 — это 344, то 80 умножить на 43 — это 3440. Вы уже поставили лишний ноль под 2, поэтому просто поставьте 344 перед 0, который вы уже поставили на шаге 3. Обратите внимание, что 3440 — это 80, умноженное на 43, и вам нужно отслеживать это до конца. справа от эшафота. Вы уже записали ноль на шаге 3, поэтому просто поставьте 8 перед этим 0. Вы только что вычли восемьдесят 43 из делимого, так как 80 умножить на 43 равно 3440. Вычтите 3440 из 3512, чтобы получить 72.

    Шаг 5: Теперь нам нужно посмотреть, сколько еще 43 можно вычесть из 72. На этот раз нет лишних нулей, как в шаге 4. Мы можем вычесть еще 1 43 из 72, поэтому запишем 1 справа от подмостей и из 72 вычтите 43, чтобы получить 29.

    Шаги 6, 7 и 8: Так как 29 меньше 43, это остаток, поэтому сложите числа справа от лески, чтобы получить частное. Запишите ответ как частное, а остаток в клеточку вверх на вершине проблемы. Проверьте ответ. Вы можете использовать калькулятор, чтобы умножить 81 на 43, что равно 3483, а затем прибавить 29..

    Последние четыре шага показаны ниже.

    Если бы вы показали свою работу, вся задача выглядела бы так:

    Вот как это будет выглядеть, если сделать это так же, как в упражнении 2:

    Для обоих методов, когда вы складываете числа справа от подмостков, чтобы получить частное, будьте осторожны, чтобы выровнять разрядные значения! Оба метода дают одинаковый результат и работают одинаково хорошо. Важно то, что процедура имеет смысл для вас!

    Еще два примера с использованием числа, кратного десяткам (с добавлением лишних нулей), показаны ниже.

    В приведенном выше примере многие люди (дети и взрослые) пропускают запись нуля в разряде десятков для ответа 306, когда они делают это так, как обычно учат. Очень распространенный ответ на эту задачу — 36 р. 21. В первом шаге, описанном выше, запись полного числа сбоку действительно подчеркивает тот факт, что вы не просто умножаете 26 на 2, чтобы получить 52, вы умножаете 26 на 9.0320 200 , чтобы получить 5200. Вы знаете, что ответ на задачу будет больше 200, поскольку вы уже вычли 200 26-х из 7977. Когда у вас осталось 177, вы просто отмечаете, что можете вычесть еще несколько 26-х. Если аккуратно записать числа справа, то в десятках не останется чисел, которые можно было бы сложить. Путаница с нулем даже не возникает!

    Упражнение 3

    Для каждой задачи на деление составьте неполную таблицу умножения до 8-кратного делителя. Затем нарисуйте каркас и используйте многократное вычитание для каждого деления. Используйте числа, кратные десяткам (при необходимости добавляя дополнительные нули), чтобы решить эти задачи. Затем проверьте ответ. Все работы должны быть показаны в отведенном для этого месте. После проверки напишите ответ на отведенном месте в начале задачи. Для проверки ответа можно использовать калькулятор, но запишите шаги.

    а. \(1230 \дел 27\) = ______________

    Частичная таблица умножения: Леса: Чек:

    б. \(603 \дел 48\) = _______________

    Частичная таблица умножения: Эшафот: Чек:

    в. \(1346 \дел 41\) = ______________

    Частичная таблица умножения: Леса: Чек:

    д. \(3512 \дел 43\) = ______________

    Частичная таблица умножения: Леса: Чек:

    эл. \(66289 \дел 325\) = ____________

    Частичная таблица умножения: Леса: Чек:

    ф. \(5222 \дел 21\) = ______________

    Частичная таблица умножения: Леса: Чек:

    Выполнение частичной таблицы умножения сбоку в сочетании с методом повторного вычитания совершенно необязательно. Но, если вы сначала составите таблицу, вам не нужно делать никаких умножений или оценок в уме. Другой вариант — полностью пропустить частичную таблицу умножения и по-прежнему использовать метод повторного вычитания. В этом случае, если вы не уверены, сколько раз можно вычесть делитель, недооценивайте на каждом шаге, что вы не совсем уверены, сколько раз делитель нужно вычесть. Другими словами, убедитесь, что вы не умножаете что-либо на делитель и не получаете число, слишком большое для вычитания из делимого. Недооценка допустима при использовании метода повторного вычитания, потому что вы просто вычитаете еще немного на следующем шаге. Например, в предыдущем примере вы могли вычесть 200 26 (5200) на первом шаге, а затем еще 100 26 на втором шаге, или вы могли вычесть 300 26 на первом шаге, умножив 26 на 3 в уме. если бы тебе было уверен, что 26 умножить на 3 не так уж и много. Мне нравится частичная таблица умножения, потому что она избавляет от догадок. В приведенном ниже примере деление выполняется с помощью многократного вычитания без составления таблицы. Вместо этого вы оцениваете, сколько делителей можно вычесть на каждом шаге. В этом случае умножение должно быть сделано в вашей голове, или на стороне, по ходу дела.

    Вот способ разделить \(8673 \div 53\) без обычной предварительной работы по составлению частичной таблицы умножения. Это означает, что умножение должно выполняться по пути. Важно отметить, что можно недооценивать при использовании метода повторного вычитания (в отличие от традиционного метода деления), потому что вы всегда можете вычесть больше на следующем шаге. С другой стороны, переоценка заставит вас вернуться и переделать задачу. Во-первых, я замечаю, что 53 входит в 86 один раз, поэтому я вычитаю 100 53, или 5300, из 8673. Затем оцените \(337 \div 53\). Поскольку я не уверен, будет ли это 5 или 6 раз, я не рискую и поэтому оцениваю 5. Итак, я вычитаю 50, умноженное на 53, или 2650, из 3373. (Обратите внимание, что я должен выполнить умножение в уме. ) Ну, я вижу, что еще 53 можно вычесть из 72, поэтому я вычитаю 10 раз 53, или 530, из 723. Тогда, я оцениваю, 53 переходит в 19.3 как минимум 3 раза, поэтому я вычитаю 3 раза 53 из 193. Поскольку 34 меньше 53, 34 — это остаток. Сложите числа сбоку, чтобы получить частное 163. Проверьте, умножив \(53 \times 163\) и добавив остаток от 34, чтобы получить 8673.

    В традиционном алгоритме деления нет места недооценке или переоценке. Вы должны вернуться и стереть, если вы поместили неправильную цифру в определенное значение разряда. Если вы оцените неправильно, умножение окажется пустой тратой времени и усилий, и вам придется начинать все сначала. Кроме того, большинство людей не задумываются о том, что и почему они помещают цифры в определенное место. Они не думают о разрядных значениях или о том, что деление на самом деле связано с повторным вычитанием. Я думаю, что метод лесов, с составлением частичной таблицы умножения или без него, способствует лучшему пониманию деления и менее напряжен, чем традиционный метод «угадывания», где абсолютно нет места для ошибки. Одно из преимуществ традиционного алгоритма состоит в том, что он является более коротким путем к методу каркаса и требует меньше места на бумаге.

    В конце концов, именно тот, кто выполняет деление, сам решает, как это сделать.

    Вот пример деления \(27984 \div 37\) методом удвоения.

    При составлении таблицы умножения она не обязательно должна быть частичной. В десятичной системе счисления удвоение, чтобы получить делитель, умноженный на 1, 2, 4 или 8, в большинстве случаев дает нам достаточно близкие оценки. Наихудший возможный случай состоит в том, что 7 — это правильное количество раз, которое можно что-то вычесть, как в приведенном выше примере. В этом случае мы сначала вычтем 4, затем 2, затем 1. Это случается не слишком часто, но время от времени случается. Другая стратегия состоит в том, чтобы добавить еще одно дополнение к частичной таблице, чтобы вы также знали, чему равен 6 делителей. Для этого, вычислив, чему равен 4 делителя, добавьте еще 2, как показано ниже (добавьте 74 вместо удвоения 148), затем снова прибавьте еще 2 (снова прибавьте 74), чтобы получить число, умноженное на 8. Посмотрите на разницу.

    На самом деле, мы могли бы использовать повторное сложение, чтобы узнать ответы делителя, умноженного на , для каждой цифры от 1 до 9, и в этом случае никогда не возникнет ни недооценка, ни переоценка. Хотя это не оставляет абсолютно никаких догадок, в большинстве случаев это больше работы, чем необходимо. Вот пример того, как кто-то может решить ту же задачу, что и на предыдущей странице, сначала составив полную таблицу умножения. Разделить легко, если вы сделаете это сначала, но в начале нужно много подготовительной работы, чтобы сделать таблицу.

    Вспомните три различных способа деления \(27984 \div 37\). Первый способ, показанный на предыдущей странице, имел короткую частичную таблицу умножения (всего три сложения для подготовительной работы), но метод повторного вычитания занимал немного больше времени. Потребовалось семь вычитаний, чтобы получить ответ (посмотрите направо от эшафота), потому что 700 сделали три шага 400, 200, 100; 50 сделали два шага 40 и 10; а 6 потребовалось два шага 4 и 6. Второй способ имел еще одно дополнение для подготовки таблицы умножения, а затем требовалось только пять вычитаний, чтобы получить ответ (посмотрите справа от леса), потому что 700 занял два шаги 600 и 100; 50 сделали два шага 40 и 10; и 6 сделали один шаг. Третий способ, показанный вверху, требовал восьми дополнений подготовительной работы для составления таблицы умножения, но деление было легким, заняв всего три шага. Если вы делаете это таким образом, вы можете также написать 756 вверху, как вы делаете это, используя традиционный алгоритм, потому что вы никогда не можете недооценивать или переоценивать.

    В зависимости от конкретной проблемы один метод может быть быстрее другого, но во многих случаях вы не узнаете об этом, пока не решите задачу. Обычно я просто удваиваю число до 8, чтобы получить достаточно хорошие оценки, но если вы хотите составить более полную таблицу, это тоже работает. Вы должны решить, предпочитаете ли вы сделать больше подготовительной работы заранее, чтобы, возможно, ускорить разделение.


    Эта страница под названием 7.2: Алгоритмы разделения распространяется под лицензией CC BY-NC 4.0 и была создана, изменена и/или курирована Джули Харланд с использованием исходного контента, который был отредактирован в соответствии со стилем и стандартами платформы LibreTexts; подробная история редактирования доступна по запросу.

    1. Наверх
      • Была ли эта статья полезной?
      1. Тип изделия
        Раздел или страница
        Автор
        Джули Харланд
        Лицензия
        CC BY-NC
        Версия лицензии
        4,0
        Показать страницу Содержание
        нет
      2. Теги
        1. источник@ https://sites. google.com/site/harlandclub/my-books/math-64

      В пемдах сложение предшествует вычитанию?

      Вопрос задан: Кристина Кин

      Оценка: 4,2/5 (15 голосов)

      Примеры применения правила PEMDAS

      Основываясь на порядке операций, умножение имеет приоритет над сложением и вычитанием, поэтому мы будем умножать первыми. Затем вычесть, а затем добавить, так как операция вычитания предшествует сложению слева направо .

      Что на первом месте: сложение или вычитание?

      Сложение и вычитание также работают вместе. Сначала вы можете выполнить вычитание , а можете сначала выполнить сложение. Они являются частью одного и того же шага, однако их можно выполнять только после элементов в скобках, показателей степени и любого умножения и деления.

      Вы делаете сложение или вычитание сначала Пемдас?

      Порядок операций — это правило, указывающее правильную последовательность шагов для вычисления математического выражения. Мы можем запомнить порядок, используя PEMDAS: скобки, экспоненты, умножение и деление (слева направо), Сложение и вычитание (слева направо) .

      Вычитание или сложение идут первыми в порядке операций?

      Порядок операций можно запомнить по аббревиатуре PEMDAS, которая означает: круглые скобки, показатели степени, умножение и деление слева направо, сложение и вычитание слева направо. Сначала , упростите скобки. Затем сделайте показатели. Далее умножайте.

      Вы добавляете перед вычитанием?

      Порядок операций говорит вам сначала выполнить умножение и деление слева направо, прежде чем выполнять сложение и вычитание. … (Обратите внимание, что сложение не обязательно выполняется перед вычитанием .)

      31 связанный вопрос найден

      Имеет ли значение порядок вычитания?

      Когда мы выполняем вычисление сложения, порядок, в котором мы складываем числа, не имеет значения . Например, 8 + 3 + 5 совпадает с 3 + 8 + 5 и дает нам тот же ответ, 16. Однако, когда мы выполняем вычитание, нам нужно особенно внимательно относиться к порядку чисел.

      Применяется ли Бодмас, если нет скобок?

      Подобные проблемы часто циркулируют в социальных сетях с надписями типа «90% людей понимают это неправильно». Просто следуйте правилам BODMAS, чтобы получить правильный ответ. Здесь нет скобок или порядков, поэтому начните с деления и умножения .

      Всегда ли применяется порядок действий?

      Всегда начинайте с операций, заключенных в круглые скобки. … В любых скобках вы следуете порядку операций, как и в любой другой части математической задачи. Здесь у нас есть две операции: сложение и умножение. Поскольку умножение всегда идет первым, мы начнем с умножения 6 ⋅ 2 .

      Можно ли вычитать числа в любом порядке, и разница останется прежней?

      Хотя значение коллекций изменено, их различие остается прежним . Операция вычитания не является ни коммутативной, ни ассоциативной. Вычитание не является коммутативным: если a и b разные счетные числа, a ½ b и b ½ a не равны.

      Что такое правило DMAS в математике?

      Деление, умножение, сложение и вычитание (DMAS) — это элементарное правило для порядка выполнения двоичных операций.

      Всегда ли сначала идет умножение?

      Порядок операций говорит вам сначала выполнить умножение и деление , работая слева направо, прежде чем выполнять сложение и вычитание. Продолжайте выполнять умножение и деление слева направо. Далее складываем и вычитаем слева направо.

      В каком порядке вы решаете уравнения?

      Чтобы помочь учащимся в Соединенных Штатах запомнить этот порядок операций, учителя вставляют в них аббревиатуру PEMDAS: круглые скобки, показатели степени, умножение, деление, сложение, вычитание .

      Бодмас и Пемдас одно и то же?

      BODMAS, BIDMAS и PEMDAS — это аббревиатуры для запоминания порядка операций в математике. BODMAS означает скобки, порядки, деление, умножение, сложение и вычитание. BIDMAS и PEMDAS делают одно и то же, но используют разные слова .

      Имеет ли значение порядок сложения и вычитания?

      Да, сложение и вычитание коммутативны : Операции можно выполнять в любом порядке.

      Какое правило сложения и вычитания?

      Правила сложения и вычитания целых чисел: 1) Если два числа имеют разные знаки, такие как положительные и отрицательные, вычтите два числа и получите знак большего числа . 2) Если два числа имеют одинаковый знак, т. е. положительные или отрицательные знаки, сложите два числа и укажите общий знак.

      В Bodmas сложение предшествует вычитанию?

      Правило BODMAS гласит, что мы должны сначала вычислить скобки (2 + 4 = 6), затем порядки (5 2 = 25), затем любое деление или умножение (3 x 6 (ответ на скобки) = 18). ) и, наконец, любое сложение или вычитание (18 + 25 = 43).

      Как вы переносите число при вычитании?

      Всегда двигаться влево

      При вычитании вы всегда будете двигаться влево. Всегда начинайте с наименьших значений. Если у вас есть пятизначное число, такое как 12 345, вы сначала начнете вычитать значения из столбца единиц. Затем вы перейдете к столбцам десятков, сотен, тысяч и десятков тысяч.

      Как называется результат вычитания?

      Формально вычитаемое число называется вычитаемым, а число, из которого оно вычитается, — уменьшаемым. Результат разница .

      Каковы правила вычитания?

      Вычитание: Измените знак вычитания на сложение и переверните знак второго числа. Затем добавьте числа. (Это определение правила вычитания или двух штрихов.)

      Всегда ли Пемдас является правилом?

      Просто, правда? Мы используем правило «порядка действий», которое выучили в детстве: «Пожалуйста, извините, моя дорогая тетя Салли», или PEMDAS, что означает скобки, показатели, умножение, деление, сложение, вычитание. * Эта удобная аббревиатура должна разрешить любые споры, но это не так, , потому что это вовсе не правило .

      Вы по-прежнему соблюдаете порядок операций без скобок?

      Объяснение: ВСЕГДА необходимо соблюдать правильный порядок операций , иначе у одного выражения могло быть несколько ответов, в зависимости от того, как это было сделано. Это явно не правильно и не разумно. В вычислениях в первую очередь делаются самые сильные операции — Степени и корни.

      Каков ответ на эту математическую задачу 50/50 25×0 2 2?

      После небольшого исследования со стороны многих сертифицированных учителей математики у нас есть правильный ответ и процесс его подтверждения. Вот, 50+50-25×0+2+2 = 104 . И снова ответ на сложную математическую задачу восьмиклассника — 104.

      Каковы четыре правила математики?

      Четыре правила математики: сложение, вычитание, умножение и деление .

      Какая скобка решается первой?

      В соответствии с правилом Бодма, если выражение содержит скобки ((), {}, ), мы должны сначала решить или упростить скобку, за которой следует (степени и корни и т. д.), затем умножение, деление, вычитание и сложение слева направо.

      Когда не следует использовать Bodmas?

      Распространенные ошибки при использовании правила BODMAS

      Ошибка возникает в некоторых случаях из-за отсутствия правильного понимания сложения и вычитания целых чисел . Например, 1-3+4 = -2+4 = 2. Но если вы упростите это так: 1-3+4=1-7= -6, вы получите неверный ответ.

      Четыре основных математических действия

      Сложение

      Сложение двух или более чисел дает нам другое число. Числа, которые складываются, называются слагаемыми, а полученное таким образом новое число называется суммой. Например,
      34670 + 12345 = 47015
      Здесь 34670 и 12345 называются слагаемыми, а 47015 называется суммой 34670 и 12345. Большие числа складываются так же, как и маленькие числа.

      Сложение путем группировки

      Когда мы складываем три числа, мы можем сначала сложить любые два числа, а затем добавить к сумме третье число. Другими словами, мы можем сгруппировать любые два из трех чисел, чтобы найти сумму трех чисел. Если нам нужно добавить более трех чисел, мы можем аналогичным образом сгруппировать любые два из заданных чисел несколькими способами и добавить их. Например, предположим, что мы хотим добавить 234523, 123098, 555623 и 876543. Мы можем найти эту сумму любым из следующих способов:

      • [(234523 + 123098) + 555623] + 876543
        = 9132444444444444444444444444444444444444444444444444444444444444444444444444444444444444444444444444444444444444444434344444444343443434434ня.
      • [(234523 + 123098) + 876543] + 555623
        = (357621 + 876543) + 555623
        = 1234164 + 555623 = 1789787

      Итак, мы можем сгруппировать их любым удобным способом, сложив три или более чисел.

      Вычитание

      Вычитание одного числа из другого числа дает нам третье число. Полученное таким образом новое число называется разностью двух чисел. Например,
      70000 – 67429 =2571
      Здесь 2571 – это разница между 70000 и 67429. Большие числа вычитаются так же, как и маленькие.

      Подробнее:

      • Целые числа и примеры
      • Фундаментальные операции на целых числах
      • Целые числа и его свойства
      • Начальные подсказки для вспоминающего свойства
      • 3333.

        Умножение

        Умножение двух или более чисел дает нам другое число. Полученное таким образом новое число называется произведением этих чисел. Например, 11 × 13 = 143.
        Здесь 143 — это произведение 11 и 13. 11 и 13 называются действующими лицами числа 143. Обратите внимание, что 1 всегда является делителем любого числа.
        Возьмем другой пример, скажем, 855 × 73 = 62415. Здесь 855 и 73 — делители 62415, а 62415 — произведение 855 и 73.
        Большие числа умножаются так же, как и маленькие.

        Умножение с использованием нулей

        Когда мы умножаем число на 10,100,1000 и т. д., мы просто помещаем эти нули справа от этого числа. Например, если мы хотим умножить 15 на 10, то ответ будет 150. Точно так же, если мы хотим умножить 15 на 100, то ответ будет 1500 и так далее. Справа от числа 15 добавлены нули.

        Деление

        Деление числа на другое число дает два новых числа — частное и остаток . Число, которое делится, называется делимым , а число, которое делится, называется делителем . Например,

        Здесь 7 — это делитель ; 66
        дивиденд ; 9 — это частное , а 3 — это остаток .

        Разделите 69205 на 432 и найдите частное и остаток.

        Здесь частное равно 160, а остаток равен 85.

        Смешанные операции с участием +, –, × и ÷

        До сих пор мы решали задачи только с одним типом операций, то есть с одним из следующих: сложение, вычитание, деление и умножение. Но что делать, если задача включает в себя две или более операций одновременно? Рассмотрим следующую задачу:
        Пример: Упростить: 16 – 6 + 2 – 3

        В первом случае ответ равен 9, а во втором случае – 5. Мы получаем разные ответы в зависимости от порядка, в котором операции выполняются. Но один из двух ответов, которые мы получили, неверен. Во избежание такой двусмысленности была принята международная конвенция.

        1. Если какое-либо математическое выражение содержит символы сложения и вычитания, мы сначала складываем, а затем вычитаем. Например, рассмотрим следующий случай: 16 – 3 + 4 – 5
          = 20 – 3 – 5    (Сложение: 16 + 4 = 20)
          = 17 – 5            (Вычитание: 20 – 3 = 17)
          = 12                  (Вычитание : 17 – 5 = 12)
        2. Если в трех операциях [(+, – , и × ) или (+, – , и ÷)], то есть кроме сложения и вычитания, в задаче присутствует умножение или деление, мы сначала умножить или разделить, а затем перейти к сложению и вычитанию соответственно. Например, рассмотрим следующие случаи.
          (a) Упрощение: 7 + 3 × 4 – 3
          В приведенном выше примере задействованы три операции: +, – и ×. Чтобы решить эту задачу, мы сначала умножаем числа, затем переходим к сложению и в конце вычитаем.
          7 + 3 × 4 – 3
          = 7 + 12 – 3 (Умножение: 3 × 4 = 12)
          = 19 – 3        (Сложение: 7 + 12 = 19)
          = 16               (Вычитание: 19 – 3 = 16)
          (b) Упрощение: 16 – 6 ÷ 2 + 8
          В приведенном выше примере задействованы три операции: +, – и ÷. Чтобы решить эту задачу, мы сначала делим, затем складываем и в конце вычитаем число. 16 – 6 ÷ 2 + 8
          = 16 – 3 + 8  (Деление: 6 ÷ 2 = 3)
          = 24 – 3        (Сложение: 16 + 8 = 24)
          = 21               (Вычитание: 24 – 3 = 21)
        3. операций, а именно, +, – , × и ÷, то существует согласованная формула, обозначаемая «DMAS», которой следуют математики. В «DMAS» D означает деление, M — умножение, A — сложение и S — вычитание. DMAS представляет порядок операций. Например, рассмотрим следующие случаи.
          (a) Упрощение: 5 + 4 × 3 – 9÷ 3
          В приведенном выше примере присутствуют все четыре операции, поэтому мы должны использовать правило DMAS, как показано ниже: 5 + 4 × 3 – 9 ÷ 3
          = 5 + 4 × 3 – 3 (Деление: 9 ÷ 3 = 3)
          = 5 + 12 – 3       (Умножение: 4 × 3 = 12)
          = 17 – 3              (Сложение: 5 + 12 = 17)
          = 14                             (Вычитание: 9 0 0 1 3) ) Упрощение: 7 × 3 – 4 + 60 ÷ 10
          В этом примере также присутствуют все четыре операции, поэтому для упрощения мы должны использовать правило DMAS. 7 × 3 – 4 + 60 ÷ 10
          = 7 × 3 – 4 + 6 (Деление: 60 ÷ 10 = 6)
          = 21 – 4 + 6      (Умножение: 7 × 3 = 21)
          = 27 – 4             (Сложение: 21 + 6 = 27)
          = 23                  (Вычитание: 27 – 4 = 3)

        Использование операции ‘Of’

        Иногда нам нужно найти значение ‘\(\frac { 1 }{ 2 }\) 16’ или ‘3 5’.
        Это означает, что нам нужно найти значение \(\frac { 1 }{ 2 }\) × 16 или 3 × 5.
        Таким образом, «из» означает умножение.
        Следовательно, \(\frac { 1 }{ 2 }\) из 16 = \(\frac { 1 }{ 2 }\) × 16
        = 8
        и 3 из 5 = 3 × 5
        = 15.
        Когда в каком-либо математическом выражении встречается операция «из», она должна выполняться перед любой другой операцией. Для решения такого рода выражений мы используем правило ODMAS, в котором O означает из, D для деления, M для умножения, A для сложения и S для вычитания.
        Рассмотрим следующие примеры.

        Пример 1: Упростить 36 ÷ 2 из 3 + 6 × 2.
        Решение: Чтобы решить это, мы сначала решим операцию «из».
        36 ÷ 2 из 3 + 6 × 2
        = 36 ÷ 6 + 6 × 2 (Из. 2 из 3 = 2 × 3 = 6)
        = 6 + 6 × 2           (Деление: 36 ÷ 6 = 6)
        = 6 + 12                (Умножение: 6 × 2 = 12)
        = 18                       (Сложение: 6 + 12 = 18)

        из 35 × 2.
        Решение: 42 ÷ 6 × 2 + \(\frac { 1}{7}\) из 35 × 2
        = 42 ÷ 6 × 2 + 5 × 2  (Из: \(\frac { 1 }{ 7 }\) из 35 = \(\ frac { 1 }{ 7 }\) × 35 = 5)
        = 7 × 2 + 5 × 2 (разделение: 42 ÷ 6 = 7)
        = 14 + 10 (умножение: 7 × 2 = 14 и 5 × 2 = 10)
        = 24 (добавление: 14 + 10 = 24)

        Использование скобок и правило BODMAS

        Рассмотрим пример, иллюстрирующий использование скобок.
        Рима купила 35 шоколадок и съела 5 из них. Оставшиеся шоколадки она распределила поровну между 6 своими друзьями. Сколько конфет она дала каждому из них?

        В этой задаче мы должны вычесть 5 шоколадок, которые съела Рима, из 35 шоколадок, которые у нее были, прежде чем разделить их между 6 ее друзьями. Итак, мы должны сначала выполнить операцию вычитания, а затем деления. В таких случаях мы заключаем в скобки ту часть, которая должна быть выполнена первой, то есть
        (35 — 5) ÷ 6 (первая решаемая скобка, т. е. 35 — 5 = 30)
        = 30 ÷ 6      (Деление: 30 ÷ 6 = 5)
        = 5
        Рассмотрим другой пример.

        Пример 3: Решите 2 из 3 × (5 + 2).
        Решение: 2 из 3 × (5 + 2)
        = 2 из 3 × 7  (Первая скобка: 5 + 2 = 7)
        = 6 × 7          (Из: 2 из 3 = 2 × 3 = 6)
        = 42              (Умножение: 6 × 7 = 42)
        Следовательно, когда задачи включают скобки, из, ×, ÷, + и –, то
        Чтобы упростить запоминание этого порядка, мы запоминаем слово BODMAS , где B — скобки, O — of’, D — деление, M — умножение, A — сложение и 9.0005 S
        для вычитания. Это называется правилом « BODMAS ».
        Иногда в числовых выражениях могут использоваться различные типы скобок.

      admin

      Добавить комментарий

      Ваш адрес email не будет опубликован. Обязательные поля помечены *